You are on page 1of 100

My Masterclass

Create exam

My history

My details

Q1.

A 22 yr old never-smoker female gym instructor who had suffered from asthma since childhood and was usually stable on
inhaled corticosteroids, long acting beta 2 agonists and as needed salbutamol presented to the Emergency Department. She had
developed symptoms of an upper respiratory tract infection three days before and had become wheezy and tight chested two
days previously. She had escalated therapy as per her asthma plan but remained wheezy.

She was given salbutamol nebulisers and IV hydrocortisone in the ambulance and appeared fairly comfortable when seen. On
examination she had saturations of 97% by pulse-oximetry and prolonged soft wheeze. Her peak flow was 150 L/min (recent
best=500). Her chest X-ray was clear and her ABG on air showed pH 7.45, paCO2 5.1 kPa and paO2 10.8 kPa.

What is the most appropriate management?

Admit to a general ward on oral steroids, nebulisers, antibiotics and oxygen


B

Admit to a monitored bed in a respiratory ward on oral steroids, nebulisers, magnesium, antibiotics and oxygen
C

Call for ICU review and admit to an ICU/HDU bed on oral steroids, antibiotics, nebulisers, magnesium and oxygen
D

Discharge home with antibiotics and inhalers as per asthma plan treatment
E

Discharge home with oral steroids, a nebuliser and 48h GP review

This is a life threatening asthma attack that has not shown adequate response to standard therapy. Peak flow of <33% of recent
best requires ICU review and management in ICU or HDU as per national BTS guidelines. It is this group of patients who is at
highest risk of suffering a fatal deterioration if discharged or admitted to unmonitored settings.

Q2.

A 67-year-old retired stonemason presented with a 9-month history of non-productive cough and gradually worsening
shortness of breath. He described no other respiratory or constitutional symptoms. He was an ex-smoker of 10 pack-years.
Examination of the respiratory system revealed bibasal fine inspiratory crepitations that did not alter on coughing. A chest X-ray
showed upper and middle zones nodules with evidence of calcification. His spirometry showed an FEV1 at 60% of predicted and
an FVC at 65% of predicted.

What is the most likely diagnosis?

A
bronchiectasis
B
Caplan’s syndrome
C
chronic obstructive pulmonary disease (COPD)
D
siderosis
E
silicosis

The clinical and radiological findings are typical for subacute silicosis. The smoking history is not substantial and the spirometry
showed a restrictive pattern, thus making COPD unlikely. Siderosis is associated with exposure to iron oxide and is commonly
associated with welding work. Caplan’s syndrome occurs in miners with seropositive rheumatic arthritis or positive serum
rheumatoid factor.

My Masterclass

Create exam

My history

My details

Q3.

You are asked to speak to a 58-year-old man and his wife on the ward. He has been admitted for routine surgery
(cholecystectomy) and when clerked says that he worked in shipyards for many years. The house surgeon enquires about
asbestos exposure, and this leads to a great deal of anxiety. His wife is very keen to know what medical risks this poses for him.
Which one of the following statements regarding asbestos exposure or asbestos-related disease is true?

adenocarcinoma is the predominant histological type in asbestos workers with lung cancer.

chemotherapy is the treatment of choice in patients with mesothelioma.


C

pleural plaques are often associated with restrictive defect in respiratory function tests.

the risk of lung cancer increases by 50-fold in asbestos workers who smoked compared to persons who neither smoked nor
worked with asbestos.

the risk of mesothelioma increases with smoking.

Asbestos exposure is associated with:

 pleural plaques
 diffuse pleural thickening
 lung fibrosis (also known as asbestosis)
 lung cancer
 mesothelioma.
The commonest abnormality seen in asbestos workers is pleural plaques, which are well-circumscribed areas of thickening
affecting the parietal pleura. Pleural plaques are even seen in patients with mild asbestos exposure. In the majority of patients,
these plaques are asymptomatic and do not cause any changes in respiratory function tests.

Mesothelioma is a tumour of the mesothlial cells or the pro-mesothelial cells. It affects the pleura and, to a lesser extent, the
pericardium and the peritoneum. The incidence of mesothelioma increases in patients with heavy asbestos exposure and in
those who were exposed to asbestos at a young age. Cigarette smoking does not increase the incidence of mesothelioma.

Mesothelioma is a fatal disease. The majority of patients die within 18 months of diagnosis. The disease is resistant to current
treatment modalities such as chemotherapy and radiotherapy. Tumor resection is indicated in only a small proportion of
patients.

Lung cancer is associated with heavy asbestos exposure. The effects of asbestos exposure and smoking are multiplicative. The
risk of lung cancer in asbestos workers who never smoke is increased by 5-fold. In smokers the risk is about 55-fold that in non-
smoking persons who never worked with asbestos.

Q4.

85 year old man was being treated on the stroke ward for acute right middle cerebral artery stroke and left basal pneumonia,
for which he was receiving co-amoxiclav. He suddenly deteriorated with acute breathlessness, but he denied any chest pain. His
vital signs included pulse 120/min, BP 165/90mmHg, temperature 37.5oC, SaO2 94% on 8 liters of O2, and respiratory rate
30/min.

On examination he had dullness and reduced air entry on the left side of the thorax. ECG revealed sinus tachycardia and CXR
showed left hemithorax white-out, with the trachea deviated towards the left.

What is the most appropriate next management step?

A
Aspirate pleural effusion
B
Chest physiotherapy
C
Enoxaparin (treatment dose)
D
Insert chest drain
E
Stop coamoxiclav; start tazocin

This patient has suffered from total collapse of the left lung, probably due to increased bronchial secretions and mucus
plugging. The initial management includes nebulisers and chest physiotherapy to try to dislodge the mucus plug and - if that fails
- bronchoscopy. He might have developed a parapneumonic pleural effusion as well, but this is not massive as the trachea is
deviated towards the side of the white-out. He is at risk of pulmonary embolus but the CXR points to another obvious diagnosis.

Q5.

A 35-year-old woman known to suffer from rheumatoid arthritis gives a history of cough with copious phlegm with intermittent
haemoptysis. She also gets recurrent chest infections. She is on weekly methotrexate along with a non-steroidal anti-
inflammatory drug (NSAID). Her chest radiograph shows linear radioluciences at both bases. A high-resolution computed
tomography scan (HRCT) of the chest confirms bronchiectasis. What is the most likely cause?

A
Cystic fibrosis.
B
Foreign body aspiration
C
Kartagener's syndrome
D
Old tuberculosis
E
Rheumatoid arthritis

Bronchiectasis may be due to many causes. Post infection (TB, measles, whooping cough etc), foreign body aspiration,
adenomas, hypogammaglobulinaemia, Kartagener's syndrome, Young's syndrome, cystic fibrosis, allergic bronchopulmonary
aspergillosis are the main causes. Up to 4 % of patients with rheumatoid arthritis develop bronchiectasis.

Q6.

A 77-year-old man with emphysema was referred to a chest clinic for consideration of oxygen therapy. Despite maximal
treatment with bronchodilators, his exercise tolerance was reduced to about 20-30 yards. Physical examination and pulmonary
function tests were consistent with emphysema, and there was no evidence of heart failure. The ECG was normal and the chest
radiograph showed hyperinflated lung fields. Oxygen saturation was 91% on air, with arterial blood gases (on air) showing pH
7.4, pCO2 3.5kPa, and pO2 7.5 kPa. What would you recommend?

A
Ambulatory oxygen therapy
B
Long-term oxygen therapy
C
Overnight SaO2 monitoring
D
Repeat ABG in 3-weeks time
E
Walking test

This patient does not meet the criteria for LTOT, which is indicated when the paO2 is consistently at or below 7.3 kPa, when
breathing air during a period of clinical stability. Clinical stability is defined as the absence of exacerbation of chronic lung
disease for the previous 5 weeks. Patients without chronic hypoxaemia and LTOT should be considered for ambulatory oxygen
therapy if they show evidence of exercise oxygen desaturation (a fall of SaO2 of at least 4% to below 90%), improvement in
exercise capacity with ambulatory oxygen therapy and motivation to use the ambulatory oxygen outside the house. Assessment
should be performed on both air and supplemental oxygen with the patient blinded as to the content of the cylinder.
Q7.

Clinical scenario - A 58 year old man presented to the Medical Admissions Unit complaining of acute onset shortness of breath
and pleuritic chest pain, on a background of several days of worsening cough. He had a history of chronic obstructive pulmonary
disease. His vital signs were as follows: temperature 37.8oC, pulse 105/min, BP 138/84 mmHg, respiratory rate 18/min. There
were no obvious abnormalities on examination of his chest, apart from a very few wheezes.

What is the best test to perform next?

A
Chest radiograph
B
CT pulmonary angiogram
C
D-dimer
D
Electrocardiogram
E
V/Q scan

It is important to exclude pneumothorax in the first instance, which can be done with a plain chest radiograph, before
considering specific imaging for pulmonary embolism. The history given puts him at moderate risk for PE, and in this context, a
negative D-dimer would not exclude thromboembolism.

Q8.

A 55y old Caucasian man with a history of severe chronic obstructive pulmonary disease and chronic alcohol abuse presented
with haemoptysis. He had never travelled from the UK, and he worked as a publican. The small volume intermittent episodes of
haemoptysis were not associated with significant cough, weight loss or night sweats.

On examination he was cachectic, with a hyperinflated chest which on auscultation had scattered crackles and wheezes,
particularly in the right upper zone. A CT scan of the chest showed extensive right upper lobe fibrotic changes with some ground
glass and ‘tree-in-bud’ appearances, but there was no mass.

What is the most likely diagnosis?

A
Fungal Infection
B
Lung Cancer
C
Mycobacterium tuberculosis infection
D
Non-tuberculous mycobacterial Infection
E
Pneumonia

This presentation, in the context of alcohol abuse, COPD and malnutrition in the absence of high risk MTB exposure, makes non-
tuberculous mycobacterial Infection the most likely diagnosis. Tree-in-bud changes on CT are highly suggestive.

Q9.

A 75-year-old man presented with anorexia and haemoptysis. He was a smoker of 40-pack-year history. Examination revealed
signs consistent with a right pleural effusion. A diagnostic aspiration was performed on the pleural effusion.

Which feature would indicate an exudative pleural effusion?

A
pleural fluid lactate dehydrogenase (LDH): serum LDH ratio >0.5
B
pleural fluid LDH more than two-thirds of the value of the upper limit of normal serum LDH
C
pleural fluid protein:pleural fluid LDH ratio >0.5
D
pleural fluid protein:serum protein ratio >0.4
E
pleural fluid protein of 29 g/L

Light’s criteria states that a pleural effusion is exudative when any one of the following conditions is fulfilled:
• pleural fluid LDH:serum LDH ratio >0.6

• pleural fluid protein:serum protein ratio >0.5

• pleural fluid LDH more than two-thirds of the value of the upper limit of normal serum LDH.

Q10.

A 74-year-old man was treated in hospital for exacerbation of Chronic Obstructive Pulmonary Disease (COPD). His condition has
improved significantly and he is keen to go home. His repeated arterial blood gas (ABG) reading (on air) showed a pH of 7.35,
pCO2 of 4.5kPa, pO2 of 7.1kPa, HCO3 of 26 mmo/L. What action would be most appropriate?

A
Discharge the patient and advise a follow up by his GP.
B
Discharge the patient and arrange a follow up in 6 weeks' time with a repeated ABG.
C
Discharge the patient once his ABG returns to normal.
D
Discharge the patient, prescribing an oxygen cylinder.
E
Request an oxygen concentrator and discharge the patient once an oxygen concentrator is fitted.

Long-term oxygen treatment is recommended if pO2 is less than 7.3kPa on two occasions in remission of COPD. This patient's
p02 is likely to improve when he recovers from his current exacerbation and his ABG should be re-checked in about 6 weeks
time. As he seems able to tolerate his mild hypoxia, he would not require any oxygen supplementation at home at this stage.

Q11.

A 22-year old Nigerian office worker presented to the Emergency Department with a 1-day history of a runny nose, sore throat,
cough and fever. In the hour before admission he had developed severe pleuritic chest pain. Vital signs included T37.5oC, pulse
110/min, respiratory rate 22/min, BP 120/74 mmHg.

What is the most appropriate first line treatment?


A
Amoxicillin (intravenous)
B
Blood transfusion
C
Diclofenac (oral or IM)
D
Low molecular weight heparin (treatment dose)
E
Oxygen (high flow)

The diagnosis is a Sickle cell crisis, precipitated by an upper respiratory infection. Sickle cell crises can be precipitated by
infections and hypoxia in those with sickle cell disease. In patients with sickle cell trait, crises are rare, but can occur in the
setting of severe hypoxia or septicaemia. Management of crises depends on adequate oxygenation, aggressive volume
replacement and analgesia (usually opiate). High dose intravenous antibiotics should be administered. Chest syndrome with
hypoxia, neurological symptoms or priapism are indications for exchange blood transfusion, with the aim of reducing the
haemoglobin S to <30%.

My Masterclass

Create exam

My history

My details

Q12.

A 60 year old woman presented with shortness of breath. The breathlessness was particularly troublesome when in bed at
night, and she had taken to using 4-5 pillows. In the past she had been keen on swimming, but now found that she got
breathless on getting into the water, and she had to stop her visits to the swimming pool. On direct enquiry she said that she
tends to have a headache on waking in the morning. There was no significant past medical history.

Which would be the most useful investigation?

Echocardiogram
B
Home peak flow monitoring
C

Lying and standing vital capacity measurement


D

Nocturnal polysomnography
E

Right heart catheterization

These symptoms of breathlessness on lying flat and breathlessness in the water suggest bilateral diaphragm palsy. In the water
pressure on the abdomen allows a paralysed diaphragm to push up in to the thorax in the same way as the abdominal contents
push the diaphragm up on lying flat. If the patient can lie flat, then watch for paradoxical abdominal movement during
respiration: the movement is inwards during inspiration in bilateral diaphragm palsy.

Q13.

A 72 year old woman presented with a history of a cough for 6 weeks. She was a former smoker with a 40 pack year history. She
continued to lead an active life, but due to her cough had been unable to sing in her church choir over the last few weeks. Lung
function tests, including a flow volume loop (see image - black lines predicted, red lines actual) were performed.

What is the most likely diagnosis?

See image

A
Asthma
B

COPD
C

Fixed airway obstruction


D

Variable extrathoracic airway obstruction


E

Variable intrathoracic airway obstruction

The flow volume loop with a flattened inspiratory limb is typical of variable extra-thoracic airway obstruction. During fixed
obstruction, both limbs would show reduced flow. Variable intrathoracic obstruction typically shows flattened expiratory limbs
due to increased intra-thoracic pressure narrowing airway lumens. Variable extrathoracic obstruction presents with a flattened
inspiratory limb as obstructing elements are pulled in by negative pressure in the airway.

Q14.

A 65-year-old man was admitted to the acute medical unit with an exacerbation of COPD. An arterial blood gas (ABG) test taken
on 2 L per minute oxygen delivered by nasal cannulae showed PO2 7.9 kPa (normal range 11.3–12.6), PCO2 11.5 kPa (normal
range 4.7–6.0), pH 7.29 and serum bicarbonate 32 mmol/L (normal range 21–29).

He was treated with nebulised bronchodilators and prednisolone, and started on non-invasive ventilation (NIV) with inspiratory
positive airway pressure (IPAP) 12 cmH2O, expiratory positive airway pressure (EPAP) 4 cmH20, with 5 L per minute of oxygen
entrained into the inspiratory circuit. The mask fitted well with minimal leak, and there was no apparent patient–ventilator lack
of synchrony.

A repeat ABG test in 1 hour showed PO2 8.5 kPa, PCO2 11.6 kPa, pH 7.30 and serum bicarbonate 31 mmol/L.

What is the most appropriate intervention now?

A
increase the EPAP to a target of 10 cmH2O as tolerated
B
increase the IPAP and EPAP to a target of 20 cmH2O and 10 cmH2O, respectively
C
increase the IPAP to a target of 20 cmH2O as tolerated
D
start an aminophylline infusion
E
transfer to the intensive care unit for intubation and mechanical ventilation

In this case there has been minimal improvement in the arterial blood gas after starting non-invasive ventilation (NIV). Good
medical therapy has already been given and there is no mask leak or patient asynchrony. The NIV settings in this case are sub-
therapeutic and the inspiratory positive airway pressure (IPAP) should be increased towards a target of 20 cmH2O in order to
improve minute ventilation and CO2 clearance. Intravenous aminophylline could be considered, but this patient will gain the
most therapeutic benefit from optimising the NIV settings.

Q15.

A 23-year-old man presented to the Emergency Department with a one day history of shortness of breath. He had no relevant
past medical history, but gave a family history of pneumothorax. A chest X-ray revealed a left-sided pneumothorax measuring
6cm at the level of the hilum.

What is the correct management?

A
Admit for observation
B
Aspiration
C
Discharge home with outpatient follow-up
D
Intercostal drain insertion
E
Surgery

The BTS guidelines for the management of primary spontaneous pneumothorax (PSP) state that patients who are breathless
and/or present with a pneumothorax >2cm at the level of the hilum should proceed to needle aspiration up to 2.5L. If this is not
successful (still breathless and >2cm), only then should a chest drain be inserted. 10% of all PSP cases have a genetic basis.
Q16.

A 19-year-old university student presented as an emergency to hospital with left-sided pleuritic chest pain and breathlessness.
She had no past medical history and took no regular medication. She smoked 20 cigarettes daily. A chest radiograph revealed a
50% left pneumothorax.

What is the most appropriate initial management?

A
aspiration of the left pleural cavity followed by repeat chest radiograph
B
chest drain insertion and admission for observation
C
chest drain insertion and discharge home with a one-way drainage (Heimlich) valve
D
referral for thoracoscopic left pleurectomy and pleurodesis
E
smoking cessation advice

In a young, previously fit person pleural aspiration may be all that is required to relieve symptoms and is the initial treatment of
choice. Surgery is usually reserved for patients with recurrent pneumothorax. Smoking cessation advice, while highly relevant, is
not the most appropriate initial management option.

My Masterclass

Create exam

My history

My details

Q17.

A 55-year-old man with insulin-dependent diabetes mellitus and chronic obstructive pulmonary disease presents with five days
of worsening breathlessness, fever and cough. On examination he looks very unwell, with a temperature of 38.7ºC, a respiratory
rate of 18/min and widespread coarse crackles and wheezes in his chest. A finger prick test reveals a glucose level of 18 mmol/l.
His arterial blood gases (breathing air) are as follows: pH 7.30, pO2 7.8 kPa, pCO2 6.8 kPa, base excess –2 mmol/l. Which of the
following is the most likely explanation for these findings?

He has a metabolic acidosis with respiratory compensation.

He has type 1 respiratory failure and significant metabolic acidosis.

He has type 1 respiratory failure.

He has type 2 respiratory failure with compensatory metabolic alkalosis.

He has type 2 respiratory failure.

Type 1 respiratory failure is defined as pO2 <8 kPa with pCO2 <6.5 kPa.

Type 2 respiratory failure is defined as pO2 <8 kPa with pCO2 >6.5 kPa.

The base excess is a calculated value that provides a measure of the degree of ‘metabolic’ as opposed to ‘respiratory’
disturbance of pH. A normal value is between –2 and +2. If this man had metabolic acidosis as a result of his diabetes, then a
significantly negative ‘base excess’ would be expected.

My Masterclass

Create exam

My history

My details

Q18.

A 61 year old man was admitted with an exacerbation of his long standing chronic obstructive pulmonary disease (COPD). He
made a good recovery but his exercise tolerance remained limited to 150 yards, and FEV1 was <30% predicted. His medications
were optimised, and the possibility of enrolling in pulmonary rehabilitation was discussed.

Pulmonary rehabilitation in patients with COPD has been shown to:

improve exercise tolerance


B

improve lung function


C

reduce disease progression


D

reduce mortality
E

reduce risk of further hospitalization

Pulmonary rehabilitation, as an adjunct to standard medical therapy, improves exertional dyspnea and exercise ability in COPD.
Favourable outcomes include increases in maximal exercise tolerance, peak oxygen uptake, endurance time during submaximal
testing, functional walking distance, and peripheral and respiratory muscle strength. Pulmonary rehabilitation has no significant
effects on FEV1 and does not attenuate the decline of airflow limitation in COPD or affect long-term survival.

Q19.

A 69-year-old man with chronic obstructive pulmonary disease (COPD) on long-term oxygen therapy (LTOT) was reviewed in
clinic. He complained of persistent leg swelling. His arterial blood gases, carried out by COPD nurses 3-weeks ago on a
supplemental oxygen flow rate of 2 l/min, showed pH 7.34, pCO2 5.6kPa, pO2 8.0kPa and HCO3 28 mmol/l. What would you
recommend?

A
Chest radiograph
B
Diuretic
C
ECG
D
Echocardiogram
E
Overnight SaO2 monitoring
E

The presence of persistent oedema or secondary polycythaemia suggests that the correction of overnight SaO2 may be
inadequate. To exclude nocturnal hypoxaemia an overnight SaO2 monitoring should be carried out.

Q20.

A 40-year-old female is breathless and has a full set of lung function performed: FEV1 1.1L (60%) FVC 1.3 L (55%) FEV1/FVC ratio
= 84% TLC = 65% predicted RV = 58% predicted TLCO = 55% predicted KCO = 105% predicted What is the most likely diagnosis?

A
Asthma
B
COPD
C
Fibrosing alveolitis
D
Obesity
E
Obliterative bronchiolitis

The lung function tests show a significant restrictive defect. Only obesity or a fibrosing disease may fit this picture given the
normal/high KCO (i.e. after correcting for alveolar volumes). The most likely answer is obesity, as the gas exchange after
correcting for the alveolar volume (TLCO) would tend to be low in fibrosing alveolitis. Both COPD and obliterative bronchiolitis
show an obstructive picture but with low TLCO whereas asthma will also produce an obstructive pattern but with normal or high
TLCO.

Q21.
An eighty three year old man was admitted with an acute confusional state. Limited history was available, but his
daughter explained that he was normally well for his age, and that his long term urinary catheter had been replaced two
days previously. He was febrile (38.9ºC), breath sounds were reduced over his left chest, and the urine in his catheter bag
was frankly purulent. A chest X-ray was performed.

What is the likely cause of the abnormality on his chest X-ray?

See image

Congenital abnormality
B

Lung cancer
C

Mesothelioma
D

Pneumonia
E

Therapeutic measure
E

He has had a thoracoplasty - surgical removal of several rib bones from the chest wall in order to collapse a lung was an
important method in the treatment of pulmonary tuberculosis before the advent of anti-tuberculous chemotherapy.

My Masterclass

Create exam

My history

My details

Q22.

A 20 year old woman with cystic fibrosis had had three admissions with respiratory infections in the last 12 months. She
produced purulent sputum daily and was finding it difficult to cough this up. Her spirometry showed an FEV1 of 50% predicted
and FVC 70% predicted. Her current treatment was postural drainage and physiotherapy in the mornings, pancreatic
supplements, vitamins, inhaled salbutamol, and annual influenza vaccination.

Which would be the most appropriate addition to her treatment?

Inhaled long acting bronchodilator

Nasal corticosteroids

Nebulised recombinant DNase

Oral amoxicillin

Oral steroids

C
Nebulised recombinant DNase can enhance sputum clearance and improve lung function, but it is expensive and its benefit
should be assessed in each individual patient. Regular inhaled corticosteroids may be helpful in cystic fibrosis but oral steroids
should be avoided. Long term use of macrolide antibiotics such as azithromycin, or nebulised antibiotics such as colomycin or
tobramycin, may reduce the frequency of exacerbations.

Q23.

A 60-year-old man with chest pain is anxious of the effect of asbestos exposure that he might have incurred in his previous jobs.
He worked as a thermal-insulating engineer for 10 years from the age of 16. He has smoked 20 cigarettes a day since he was 16.
Which one of the following statements is correct?

A
Amosite (brown asbestos) and crocidolite (blue asbestos) are less potent carcinogens than chrysotile (white asbestos).
B
Chemotherapy is the main stay of treatment of patient with mesothelioma.
C
His asbestos exposure is a risk factor for bladder cancer.
D
His smoking increases the possibility of mesothelioma.
E
The fact that he worked with asbestos at young age is a risk factor for mesothelioma.

Asbestos exposure is known to be a risk factor for mesothelioma and lung cancer. The relationship with other cancers is not
certain. The rate of mesothelioma is increasing. This is mainly due to the long latent period between exposure and the clinical
development of the disease. Amosite and crocidolite are more potent carcinogens than chrysotile. Exposure at an early age is
another risk factor. Cigarette smoking does not increase the risk of mesothelioma.

The majority of patients with mesothelioma present with advanced disease. Chemotherapy and radiotherapy alone are not
helpful. Tri-modality treatment with surgery, chemotherapy and radiotherapy may be helpful in selected cases.

Q24.

A 65-year-old smoker presents to A&E with a cough productive of green sputum and blood, fever, right-sided pleuritic
pain and breathlessness. He is confused, has a temperature of 39.8C, BP 120/70, pulse 120 regular, respiratory rate 20.
His cardiovascular examination is normal and he has bronchial breath sounds at the right apex. His full blood count, urea
and electrolytes and liver function tests are normal, excepting a leukocytosis of 16. His chest radiograph is shown (see
image). Which one of the following statements is true?
See image
A

Because of the lobar distribution, it can be assumed that the infecting organism is unlikely to be an atypical one.

He has non-severe pneumonia.

He requires a bronchoscopy to rule out cancer causing an obstructive pneumonia and haemoptysis.

He should be given full dose anticoagulation to cover the possibility that this may be a pulmonary embolus causing
pulmonary infarction and haemoptysis.

If his symptoms and signs resolve rapidly he need not have a follow-up chest radiograph.

It is not possible to predict the organism from radiographic features.

Pneumonia itself can cause haemoptysis and bronchoscopy should only be considered if pneumonia or haemoptysis fails
to resolve at follow-up.

Severity of pneumonia is defined according to the (new) Confusion (abbreviated mental test score <8), Urea >7 mmol,
Respiratory rate (>30) and Blood Pressure (systolic < 90, diastolic <60) (CURB) criteria. More than two of these features
suggests severe pneumonia and these patients should be assessed by a respiratory physician.

All smokers, patients over 50 or those with non-resolving symptoms or signs should have a follow-up chest film at 6
weeks.

Although pulmonary infarction should be on the differential diagnosis, the clinical probability strongly favours pneumonia.
There are no atypical features incompatible with pneumonia. There is lobar consolidation and he is coughing green
sputum. There are no cardiovascular features to suggest PE. He should receive regular DVT prophylaxis.

Q25.

A 27-year-old man from Thailand, who arrived in the UK 3 months ago, presented with a 6 week-history of being generally
unwell. He denied any cough or sputum production but admitted to some weight loss. His Chest X ray showed mediastinal
lymphadenopathy. What would be the most appropriate investigation to arrange next?

A
CT chest scan
B
Heaf test
C
Lung function tests
D
Lung PET scan
E
Sputum microscopy and culture

A tuberculin test would be the best option in his case as Tuberculosis is very high on the list of differential diagnoses. If this is
strongly positive, anti-TB treatment should be started, with an invasive test (mediastinoscopy) considered if there is no
response to treatment at 2 months. If the Heaf test were negative, then other investigations (including HIV testing and
mediastinoscopy) would be appropriate.

Q26.

A 60-year-old man, asymptomatic from the chest point of view, had a chest X-ray performed (see image) as a routine test
before surgery for a large inguinal hernia that was to be repaired under general anaesthesia.

What is the most likely cause of the abnormality seen?


See image
A

Chickenpox pneumonia
B

Exposure to an occupational hazard


C

Hyperparathyroidism
D

Pulmonary tuberculosis
E

Trauma

Pleural plaques are areas of fibrosis, which are often partially calcified. Although pleural plaques do not normally cause any
symptoms they are an indication of previous exposure to asbestos. They are found on the parietal pleura overlying the
posterolateral chest wall and the diaphragm (both diaphragms are involved in this patient). The pleura overlying the
mediastinum and costo-phrenic sulci are usually spared. Asbestos-related thoracic diseases include benign pleural effusion,
pleural plaques, diffuse pleural thickening, rounded atelectasis, asbestosis, mesothelioma, and lung cancer.
Q27.

A 74 yr old man with long standing chronic bronchitis presented with ankle oedema. His medications consisted of salbutamol
inhaler as needed, beclometasone inhaler bd, salmeterol inhaler bd, lisinopril, bendroflumethiazide, simvastatin and aspirin. On
examination he was cyanosed, with pulse 80/min and regular, BP 148/82 mmHg, an elevated JVP, heart sounds that were
difficult to hear because of added breath sounds, and widespread polyphonic wheezes in the chest. An echocardiogram was
technically difficult to do but showed good left and right ventricular function.

Which diagnosis is most likely?

A
Chronic renal failure
B
Congestive cardiac failure
C
Cor pulmonale
D
Drug-induced oedema
E
Nephrotic syndrome

Congestive cardiac failure is unlikely with a normal echocardiogram. Some drugs, e.g. amlodipine and other calcium channel
blockers, may cause oedema, but none of the medications that this man is on are likely causes. There is no particular reason to
expect renal failure or the nephrotic syndrome in this clinical context. Cor pulmonale by far the likeliest diagnosis.

Q28.

A 65-year-old woman presented to the medical assessment unit with a four week history of a hoarse voice. She did not have any
relevant past medical history, but was a lifelong smoker. Chest x-ray revealed no abnormalities.

What should be the next management step?

A
CT chest and neck
B
Give a course of antibiotics
C
Laryngoscopy
D
Lung function testing
E
Upper gastrointestinal endoscopy

Hoarseness that persists for more than 3 weeks must be considered to be laryngeal carcinoma until proven otherwise. If a chest
x-ray shows an abnormality, then urgent referral to the respiratory team is required. If the chest x-ray is normal, then urgent
referral to ENT for visualisation of the airways is the next step, with CT chest/neck as a second-line investigation.

Q29.

A 74 year-old man with a history of diabetes mellitus, ischaemic heart disease and chronic obstructive pulmonary disease
(COPD) was admitted with exertional dyspnea, orthopnea and bilateral pedal edema. His regular medication for COPD consisted
of tiotropium, a long acting beta-agonist, and inhaled corticosteroid. His diabetes was well controlled on gliclazide. A chest
radiograph showed evidence of mild pulmonary oedema and an echocardiogram revealed a left ventricular ejection fraction of
35%.

Introduction of a beta blocker would:

A
be contraindicated by the combination of COPD and diabetes
B
be likely to improve his life expectancy
C
be likely to reduce his life expectancy
D
be likely to result in a significant increase in frequency of COPD exacerbations
E
require reversibility testing prior to initiation

Cardiovascular disease is the leading cause of mortality in patients with COPD. Despite multiple studies supporting the benefits
of beta-blockers in this patient group, less than 50% of COPD patients receive optimal treatment on a long-term basis with
maximum tolerated doses. In particular elderly patients and those with diabetes mellitus and COPD could benefit the most from
the use of beta-blockers.
Q30.

A 43-year-old woman with long-standing asthma is admitted with an exacerbation. She is cyanosed and unable to speak more
than three words at a time. She is using her accessory muscles, chest expansion is reduced but the same on both sides, and a
wheeze can be heard bilaterally. Which one of the following is the best initial treatment?

A
Maximum inspired oxygen by face mask; decompress both sides of the chest by inserting venflons into the second intercostal
spaces in the mid-clavicular line bilaterally.
B
Maximum inspired oxygen by face mask; nebulised salbutamol (5 mg) driven by oxygen
C
Nebulised salbutamol (5 mg) driven by air
D
Nebulised salbutamol (50 mg) driven by oxygen
E
Oxygen 35% by face mask; nebulised salbutamol (10 mg) driven by 35% oxygen

Resuscitation is the first priority. Maximum inspired oxygen should be given by facemask: this is best achieved using a reservoir
bag at a flow rate of 15 l/min, which can generate an FiO2 of about 85%. Nebulised salbutamol (5-10 mg) driven by oxygen
should be given, and many would add ipatropium bromide (Atrovent, 500 microg) to the nebuliser chamber at the same time as
the salbutamol. If the woman does not improve, then call for assistance from ICU sooner rather than later.

Although it is always important to consider pneumothorax in any breathless patient, there is no evidence at all to suggest that
this woman has bilateral pneumothoraces and she would not be well served by chest decompression.

Q31.

Six weeks after an episode of right upper lobe pneumonia, a 70-year old woman had a follow up chest X-ray (see image). What
would be the correct course of action?
See image

Arrange bronchoscopy
B

Arrange chest CT scan


C

Arrange percutaneous lung biopsy


D

Insert right chest drain


E

Reassure and discharge

Right upper lobe consolidation has resolved completely. A mild increase in shadowing in the left lower lobe is due to a soft
tissue breast shadow. There are radiological signs of previous right mastectomy.
Q32.

An 18-year-old woman with asthma was seen in the emergency department with a 24-hour history of breathlessness and
wheeze. She had a history of poorly controlled asthma and took her inhaled therapy intermittently.

On examination she was dyspnoeic and distressed at rest. Her SpO2 (peripheral capillary oxygen saturation) was 89% on air
(normal range 94–98%), improving to 95% on 15 L per minute oxygen via a non-rebreathe mask. Her heart rate was 130 beats
per minute and her breath sounds were quiet on auscultation of her chest. She had been given nebulised salbutamol and
ipratropium, intravenous magnesium, and she had been started on an intravenous aminophylline infusion. An arterial blood gas
after treatment on 15 L per minute oxygen showed PaO2 9.1 kPa (normal range 11.3–12.6), PaCO2 6.0 kPa (normal range 4.7–
6.0) and pH 7.35, with serum total bicarbonate 20 mmol/L (normal range 21–29).

Which intervention is most appropriate at this stage?

A
admission to the intensive care unit
B
give a further dose of intravenous magnesium
C
start an intravenous ketamine infusion
D
start an intravenous salbutamol infusion
E
start non-invasive ventilation

This patient has near fatal asthma as defined by the high-normal PaCO2 on analysis of her arterial blood gas. She should be
admitted to the intensive care unit as it is likely she will need intubation and mechanical ventilation having already received
maximal medical therapy. Intravenous salbutamol and a repeated dose of intravenous magnesium are unlikely to provide any
additional bronchodilation over and above the treatment she has already received. Intravenous ketamine has a bronchodilator
effect but should not be used outside an intensive care setting. Non-invasive ventilation should not be used in this situation
because of the risk of worsening hyperinflation.
My Masterclass

Create exam

My history

My details

Q33.

A 44-year-old woman was referred to the respiratory outpatient service with a 6-week history of worsening breathlessness on
exertion and noisy breathing. She had previously been extremely fit and was a competitive triathlete. She had had a melanoma
removed from her leg 4 years previously. There was a family history of asthma in her mother and hay fever in her younger
daughter. She had never smoked. On physical examination the cardiovascular system and respiratory systems were normal at
rest. The results of spirometry and a flow–volume loop were as follows:

• FEV1 3.1 L (104% predicted)

• FVC 3.6 L (106% predicted)

• peak expiratory flow rate 350 L/min (85% predicted)

• in the flow–volume loop expiratory flow is shown as positive and inspiratory

flow as negative (Fig 42).

What is the most likely cause of her breathlessness?


See image

asthma
B

extrathoracic compression of the trachea


C

intrathoracic large airway obstruction


D

pleural effusion
E

tracheobronchomalacia

B
The flow–volume loop shows normal expiratory flow but significant limitation of inspiratory flow: the inspiratory graph should
be semi-circular and concave upwards, whereas it is considerably flattened in this example. In addition, the mid-inspiratory flow
should be approximately equal to the mid-expiratory flow of 4 L per second, whereas here it is approximately 2 L per second.
Extrathoracic large airway narrowing is typically worse on inspiration, whereas intrathoracic large airway obstruction (for
example, due to tracheobronchomalacia or other cause) is worse on expiration. The flow–volume loop findings and spirometry
are not suggestive of asthma or pleural effusion.

Q34.

A 72-year-old man presented with sudden onset severe epigastric pain, not relieved by vomiting, or by a GTN spray. His past
medical history included ischaemic heart disease and ulcerative colitis. On examination his abdomen was soft, with slight
epigastric tenderness, but there was no rebound and bowel sounds were present. The ECG showed sinus tachycardia and right
bundle branch block (RBBB). A chest radiograph showed bilateral atelectasis. Breathing air, his oxygen saturation was 86%, and
arterial blood gas analysis revealed pH 7.42, p02 6.8kPa, pCO2 6.5 kPa. What is the most likely diagnosis?

See image

Aortic dissection
B

Myocardial infarction
C

Oesophageal rupture
D

Pneumonia
E

Pulmonary embolism
E

Severe hypoxia accompanied by tachycardia, RBBB and bilateral atelectasis strongly suggest pulmonary embolism. None of the
other diagnoses could explain all of these findings.

Q35.

A 77-year-old man is admitted as an emergency with increasing shortness of breath and a productive cough with green purulent
sputum. His chest radiograph shows hyperinflation but no consolidation. Arterial blood gases shows an acute respiratory
acidosis. Which of the following statements is true?

A
Beta-adrenergic agonists are superior to anticholinergic agents.
B
Bronchodilators in the acute exacerbation have no effect on forced vital capacity.
C
Metered dose inhalers are more expensive than nebulisers.
D
Oxygen should not be given if he has significant hypercapnia.
E
There are fewer side effects with ipratropium bromide than with beta adrenergic agonists.

Oxygen should be given for exacerbation of chronic obstructive pulmonary disease, aiming for oxygen saturation values of 90-
92% with corresponding partial pressures of arterial oxygen of 60-65 mmHg. These targets ensure acceptable haemoglobin
saturation whilst reducing the liklihood of the hypercapnia that can accompany the use of supplemental oxygen.

Bronchodilators increase the FEV1 and forced vital capacity (FVC) by 15 to 29% over 60 to 120 minutes.

Beta adrenergic agonists are often used as first-line treatment for acute exacerbations of chronic obstructive pulmonary
disease, but have not been shown to be superior to anticholinergic agents and the two classes of agent are often used in
combination. Side effects are less frequent and milder with ipratropium bromide than with beta agonists.

Metered dose inhalers are cheaper than nebulisers.


Q36.

An 82-year-old woman was reviewed in the respiratory outpatient clinic. She had a past medical history of COPD and peripheral
vascular disease. She was receiving inhaled treatment with tiotropium and budesonide/formoterol. Her post-bronchodilator
spirometry showed FEV1 38% predicted, FVC 87% predicted and an FEV1:FVC ratio of 0.45.

According to the GOLD (Global Initiative for Chronic Obstructive Lung Disease) classification, what is the severity of her airflow
limitation?

A
mild
B
moderate
C
severe
D
unable to tell from the above information
E
very severe

The GOLD (Global Initiative for Chronic Obstructive Lung Disease) classification of airflow obstruction in COPD is based upon the
post-bronchodilator FEV1:

• mild – FEV1 ?80% predicted

• moderate – FEV1 ?50% to <80% predicted

• severe – FEV1 ?30% to <50% predicted

• very severe – FEV1 <30% predicted.

Q37.
A 45 year old man who worked as an HGV driver presented to the respiratory clinic with a 6 month history of waking up
breathless during the night. He did not complain of wheeze or cough at any time. His partner reported that he sometimes
snored. There was a past medical history of asthma (well controlled with 2 puffs of beclomethasone BD), hypertension
(ramipril 5mg od) and hypercholesterolaemia (simvastatin 40mg on). His salbutamol inhaler did not help his night-time
breathlessness. On examination he had a BMI of 35 and blood pressure 130/82 mmHg. His Epworth score was 9/24.

What is the single most important test?

A
CT pulmonary angiogram
B
Echocardiogram
C
Metacholine challenge test
D
Overnight oximetry
E
Pulmonary function tests

This man is an overweight HGV driver. The night-time waking could point to obstructive sleep apnoea. Although his Epworth
score is not significantly raised it is important to rule out OSA as it has significant implications for his job and is curable with
CPAP. Although an echocardiogram and lung function tests are important normal blood pressure and lack of other asthmatic
symptoms make them less urgent. There is no indication for a metacholine challenge test in this situation.

Q38.

A 27 year old woman was admitted with worsening asthma due to a viral infection. Two days later, after an initial improvement
with corticosteroids and nebulised bronchodilators, she became severely dyspneic, tachypneic, hypotensive and tachycardic.
Auscultation of the chest revealed bilateral fine crackles. A chest X-ray showed extensive, bilateral patchy lung infiltrates.
Arterial blood gas analysis on FiO2 0.6 demonstrated pH 7.5, PaO2 6.9 kPa, PaCO2 2.1 kPa, HCO3 22.3. mmol/l.

What is the most appropriate treatment?

A
Furosemide (intravenous)
B
Intubation and ventilation
C
Isosorbide (intravenous)
D
Salbutamol (nebulised)
E
Vancomycin (intravenous)

This patient has developed acute respiratory distress syndrome (ARDS) and requires ITU admission. The diagnosis of ARDS is
based on the 2012 “Berlin” criteria which relies on history of acute lung injury within 1 week of apparent clinical insult, chest
radiograph demonstrating bilateral airspace shadowing, exclusion of volume overload/cardiac failure and arterial blood gas
analysis (PaO2/FiO2 ratio-mild ARDS ≤ 39.9 kPa, moderate ARDS ≤ 26.6 kPa and severe ARDS ≤ 13.3 kPa). Despite improvements
in ventilation strategy, the mortality from ARDS approaches 40%.

Q39.

You have been asked to see a 56-year-old gentleman in A&E. He was brought in with type I respiratory failure, has already
been intubated and is waiting to be transferred to ITU. His wife says that he is quite a fit man and is not on any
medication. He developed cough, fever, headaches and body aches 1 week ago. Since the last 24 hours he has developed
dyspnoea on exertion. His wife found him unresponsive today. He is a car salesman by profession and keeps racing
pigeons. His wife also mentions that two of his favourite pigeons died 2 weeks ago.

Examination is consistent with a pneumonic illness. He has mild splenomegaly and a pale macular rash on the abdomen,
anterior chest wall and legs. Which of the following is the likely causative organism?

A
Chlamydia psittaci
B
Haemophilus influenzae.
C
Legionella pneumophila
D
Mycoplasma pneumoniae
E
Staphyloccocus aureus
A

Psittacosis is a systemic illness with primarily pulmonary manifestations. The diagnosis is suggested by exposure to pigeons. The
rash (Horder's spots) and splenomegaly are characteristic. Diagnosis can be made by a four-fold increase in the complement-
fixing antibodies. Tetracycline for 2 weeks is the treatment of choice.

Q40.

The image shows the flow volume loop of a patient with exertional dyspneoa. The diagnosis is:

See image

asthma

emphysema

fixed upper airway obstruction


D

restrictive lung defect

variable extrathoracic obstruction

The marked limitation of both inspiratory and expiratory flow rates is characteristic of fixed upper airway obstruction, either
intra- or extra-thoracic.

Q41.

A 35-year-old man presented to the Emergency Department with a three-day history of breathlessness, chest pain and
fevers. A chest radiograph revealed extensive right-sided consolidation and a chest ultrasound showed a moderate pleural
effusion. He was started on amoxicillin and clarithromycin. A pleural aspirate showed pH 7.1, with the remaining results
still pending.

What is the correct management?

A
Add vancomycin
B
Arterial blood gas analysis
C
Await Gram stain of pleural aspirate
D
Insert intercostal drain
E
Repeat chest x-ray

D
The BTS guidelines for the management of pleural infection state that patients with the following results should receive prompt
pleural space chest tube drainage: 1) frankly purulent pleural fluid; 2) pleural pH<7.2; 3) presence of organisms identified by
Gram stain or culture.

Q42.

A 38-year-old man presented acutely with pyrexia and a productive cough. His past medical history included renal
transplantation, and he had been treated by his general practitioner with levofloxacin with no effect. Examination of the
respiratory system revealed bilateral widespread coarse inspiratory crepitations. A CT scan of the chest revealed widespread
nodular shadowing with indistinct margins and lesions with halo signs. Serology showed positive galactomannan.

What is the most likely diagnosis?

A
Aspergilloma
B
Invasive aspergillosis
C
Mycobacterium avium infection
D
Mycobacterium tuberculosis infection
E
Pneumocystis Jirovecii pneumonia

The clinical picture and radiological findings are typical for invasive aspergillosis. The disease is usually only seen in those who
are immuno-compromised. Galactomannan is a serological test for the disease, although it is commonly false positive, especially
after the use of piperacillin/tazobactam. Aspergilloma does not present with systemic involvement.

Q43.
You are called to the resuscitation room to see a 25-year-old man whose condition has suddenly deteriorated. He had arrived 30
minutes earlier with a 2-hour history of central pleuritic chest pain and breathlessness. He collapsed while awaiting radiograph
and now is agitated and cyanosed with pulse 120/min and BP 80/40. Oxygen saturation is reading 79%, with the patient
breathing high flow oxygen via a re-breathe mask. Respiratory examination reveals reduced breath sounds in the right lung field
with deviation of the trachea towards the left. Percussion is resonant bilaterally. What immediate course of action should you
take?

A
Arrange for urgent portable chest radiograph
B
Check arterial blood gases and commence Bipap if hypoxia is confirmed.
C
Contact ITU to arrange for the patient to be ventilated
D
Insert large bore needle into left hemithorax
E
Insert large bore needle into right hemithorax

Initially the history might suggest a number of diagnoses, including cardiac tamponade, massive pulmonary embolism,
haemothorax or aortic dissection; however the respiratory examination findings indicate that he almost certainly has sustained
a spontaneous pneumothorax, which has now developed into a tension pneumothorax.

This is now a peri-arrest situation, and there is no time to arrange for portable chest radiograph, before attempting to reduce
the pressure in the right hemithorax with the insertion of a large bore needle. If the diagnosis is correct, insertion may be
accompanied by a loud ‘hiss’.

Positive pressure ventilation is relatively contraindicated in this situation, and will probably not be required once the lung has
re-inflated.

Q44.

A 45-year-old non-smoker presented to the Emergency Department with a 2-week history of fevers, cough and productive
sputum. She had been treated with amoxicillin 250 mg tds for 7 days, but she continued to have persistent fevers. Her
o
Respiratory rate was 30/min, BP 90/60 mmHg, temperature 38 C, and oxygen saturation was 96% on air. She was admitted to a
medical ward.

Which is the most appropriate management?

A
Add clarithromycin (oral)
B
Stop amoxicillin; culture sputum and blood
C
Stop amoxicillin; start co-amoxiclav (intravenous)
D
Stop amoxicillin; start co-amoxiclav (intravenous) and clarithromycin (intravenous)
E
Stop amoxicillin; start co-amoxiclav (oral)

She has not responded to basic treatment for community-acquired pneumonia. She also has a CURB-65 score (BTS
recommended severity score) of 2, where C= confusion, U= urea > 7mmol/L, R= resp rate ≥30, B= systolic<90 or diastolic≤60
mmHg, 65= age over 65 years. She requires intravenous therapy appropriate for severe community acquired pneumonia.

Q45.

A 56-year-old man with emphysema presented to the Emergency Department with increasing dyspnoea and left-sided
pleuritic chest pain. A chest radiograph revealed a left-sided pneumothorax, with the lung edge 4 cm away from the chest
wall. The most appropriate management is:

A
Admit and repeat chest radiograph tomorrow morning
B
Aspiration
C
High flow oxygen
D
Intercostal tube drainage
E
Refer for urgent respiratory outpatient clinic appointment

D
The lung edge measurement suggests a greater than 50% pneumothorax. In the context of chronic underlying respiratory
disease such as chronic obstructive pulmonary disorder, this is best managed by admission and pleural drainage.

My Masterclass

Create exam

My history

My details

Q46.

A 58-year-old man, a smoker for many years despite repeated advice that he should stop, has chronic obstructive pulmonary
disease that is increasingly limiting his exercise capacity. You wish to conduct a trial of steroid therapy. Which of the following is
the correct way to do this and interpret the outcome?

Give oral prednisolone, 10 mg daily for 2 weeks, regarding a clear statement of subjective improvement by the patient as a
positive response if accompanied by a rise in FEV1.

Give oral prednisolone, 30 mg daily for 2 weeks, regarding a clear statement of subjective improvement by the patient as a
positive response.

Give oral prednisolone, 30 mg daily for 2 weeks, regarding an increase in FEV1 of >10% and >200 ml as a positive response.

Give oral prednisolone, 60 mg daily for 4 weeks, regarding a clear statement of subjective improvement by the patient as a
positive response if accompanied by a rise in FEV1 of >10%.

Give oral prednisolone, 60 mg daily for 4 weeks, regarding a clear statement of subjective improvement by the patient as a
positive response.

C
Steroid challenge is indicated in chronic obstructive pulmonary disease (COPD) of more than moderate severity. Standard
practice would be to give prednisolone 30 mg daily for 2 weeks, regarding an increase in FEV1 of >10% and >200 ml as a positive
response. Given the non-specific effects and many side effects of steroids, it is crucial to demonstrate functional improvement:
many patients with COPD have sustained severe complications of steroid treatment, e.g. vertebral fracture, without any
evidence that the steroids were beneficial for their chest.

Q47.

A 60-year-old woman had been diagnosed as suffering from asthma. Her initial symptoms were of non-productive cough, worse
at night, intermittent wheeze and dyspnoea on exertion. She was on high dose inhaled steroids, a long acting beta agonist and a
short acting beta agonist. One year later she still complained of nocturnal symptoms and had been referred to the chest clinic.
Her chest radiograph was as shown (see image). She had never smoked and has never kept any pets. What is the best treatment
to add?

See image

Anticholinergic
B

Leukotriene antagonist
C

Oral prednisolone
D

Proton pump inhibitor


E
Theophylline

The chest radiograph shows a hiatus hernia. The underlying cause of her nocturnal symptoms may be gastrooesophageal reflux
which can cause cough as well as intermittent wheeze. She should be started on a proton pump inhibitor and at the same time
she should maintain a peak expiratory flow rate (PEFR) record. If her PEFR are acceptable, she should be gradually weaned off
some of her extensive asthma treatment.

Q48.

A 67-year-old retired stone mason presented with a 9-month history of non-productive cough and gradually worsening
shortness of breath. He described no other respiratory or constitutional symptoms. He was an ex-smoker of 10 pack
years. Examination of the respiratory system revealed bibasal fine inspiratory crepitations which did not alter on
coughing.

A CXR showed upper and middle zones nodules with evidence of calcifications. His spirometry showed an FEV1 at 60% of
predicted and an FVC at 65% of predicted.

What is the most likely diagnosis?

A
Bronchiectasis
B
Caplan’s syndrome
C
Chronic obstructive pulmonary disease
D
Siderosis
E
Silicosis

The clinical and radiological findings are typical for subacute silicosis. The smoking history is not substantial and the spirometry
showed a restrictive pattern, thus making COPD unlikely. Siderosis is associated with exposure to iron oxide and is commonly
associated with welding work. Caplan’s syndrome occurs in miners with seropositive rheumatic arthritis or positive serum
rheumatoid factor.

Q49.

A 45yr old man who was usually very fit, had never smoked and worked as an office-based architect, presented to the
Emergency Department with intense sharp right sided chest pains. The pains had started four days previously, when he had
visited his GP, been diagnosed with pleurisy and started on a course of oral antibiotics, but his pain - although duller - persisted
and was now associated with mild breathlessness and a tight feeling in his chest.

On examination he was comfortable with peripheral saturations of 94% on air and a low grade pyrexia of 37.6oC. His right lower
chest was dull to percussion and there were bronchial breath sounds audible from the right mid-zone upwards. His chest X-ray
revealed right sided consolidation and an effusion. Blood tests showed CRP >250 mg/dL and WBC 18.1x109/L. A chest
ultrasound showed a simple effusion and guided aspiration revealed cloudy straw coloured fluid but no pus. Biochemical
analysis of the fluid showed pH 6.95, protein 46 g/L, glucose 1.5 mmol/L (serum 5.5) and LDH =550 IU/L.

What is the most appropriate immediate management?

A
Admit for IV antibiotics
B
Continue prolonged course of oral antibiotics
C
Contrast CT chest
D
Insert an intercostal chest drain
E
Refer for bronchoscopy

In the setting of a simple, as yet, non-loculated early empyema, a 12F Seldinger drain placed under USS guidance suffices to
drain the fluid and is the first priority in managing this condition. Prolonged intravenous antibiotics can then be given either in
hospital or via an ambulatory or home service. Resolution is usually complete.

Q50.
A 56-year-old women, a smoker of 20 cigarettes daily, presents with a 6-month history of progressive shortness of breath. Her
past medical history is unremarkable apart from Raynaud’s syndrome for which she takes a calcium channel blocker. On
examination no significant abnormality is found apart from telangiectasia. Her chest radiograph shows clear lung fields,
prominent pulmonary arteries and mildly enlarged heart. Spirometry is normal, but gas transfer is reduced to 50% predicted.
What is the most likely diagnosis?

A
Congestive cardiac failure .
B
Cor pulmonale secondary to chronic obstructive pulmonary disease
C
Multiple pulmonary emboli
D
Pulmonary arterial hypertension
E
Sarcoidosis

Normal spirometry excludes chronic obstructive pulmonary disease.

Raynaud’s syndrome with telangiectasia and radiological appearances suggestive of pulmonary hypertension with impaired gas
transfer are most likely due to a vasculitic process in pulmonary circulation associated with an autoimmune rheumatic disorder.

Q51.

A 19-year-old man presented to the Emergency Department with a two day history of chest pain and mild shortness of breath.
He had no relevant past medical history. A chest radiograph revealed a right-sided pneumothorax which measured 1.8cm at the
hilum and 2.3cm at the apex.

What is the correct management?

A
Admit for observation
B
Aspiration
C
Discharge home with outpatient follow-up
D
Intercostal drain insertion
E
Surgery

The BTS guidelines on the management of primary spontaneous pneumothorax state that aspiration should only be attempted
if the size of the pneumothorax is larger than 2cm at the level of the hilum. Apical size of >3cm is used by the American College
of Chest Physicians to determine the need for intervention. This patient should be discharged home and followed-up in 2 weeks
with a repeat chest x-ray.

Q52.

A 55-year-old man was admitted with a community-acquired pneumonia. He had no significant past medical history and
was taking no regular medications. His CURB-65 score on admission was 4. He was treated with intravenous antibiotics
and transferred to the intensive care unit for haemodynamic support.

Which investigation is most indicated within the first 24 hours of treatment?

A
serum for β-glucan
B
serum for mycoplasma serology
C
sputum for acid-fast bacilli (AFB) smear and culture
D
throat swab for viral polymerase chain reaction (PCR)
E
urine for Legionella and pneumococcal antigen

This man has a severe pneumonia according to his CURB-65 score. The National Institute for Health and Care Excellence (NICE)
and the British Thoracic Society (BTS) guidelines in this situation recommend blood cultures, sputum for bacterial culture, and
urine for Legionella and pneumococcal antigen analysis. Further investigations such as serology or polymerase chain reaction
(PCR) to look for atypical, viral or fungal pathogens may be indicated if he does not respond to standard antibiotic therapy.
Q53.

A 47 year old homosexual man presented to the ED with a three week history of increased breathlessness. He had returned
from a holiday in New York 4 weeks previously. He was on no regular medication. He had had asthma as a child, but had not
required treatment since the age of 17.

On examination vital signs included temperature 37.5°C, pulse 96/min and BP 100/60 mmHg, with oxygen saturation 88% on air,
improving to 98% with 8L of oxygen. A chest X-ray was performed (see image).

What is the most appropriate next step?

See image

Give intravenous co-trimoxazole


B

Give nebulized salbutamol


C

Give treatment dose low molecular weight heparin


D
Organise CT pulmonary angiogram
E

Refer for bronchoscopy

The chest radiograph shows perihilar ground glass opacities in keeping with a diagnosis of PCP. The treatment is co-trimoxazole.
Prednisolone should be added in moderate to severe disease (when pO2 is less than 9.3kPa) PEs may present similarly but the
chest x ray makes PCP more likely. There is nothing here to indicate asthma.

My Masterclass

Create exam

My history

My details

Q54.

An 85-year-old woman was admitted from the Emergency Department with breathlessness. She had been a smoker and
was managed by her GP for moderate COPD (Forced Expiratory Volume 1 = 56% predicted). On examination she was
orientated and moderately breathless with respiratory rate 25/min. Her Systolic Blood Pressure was 125mmHg.
Auscultation of her chest revealed globally diminished breath sounds, particularly anteriorly on the left side, bilateral
expiratory wheeze and inspiratory crackles at the left base.

Arterial Blood Gas on air revealed pH = 7.29, pCO2 = 6.39 kPa, pO2=7.45 kPa, oxygen saturation 92% and lactate 2.1
mmol/L. Other test results included White Cell Count 16.7 ×10&sup9;/L, C-Reactive Protein 47 mg/dL (elevated) and
DDimer 745 (elevated). A CT Pulmonary Angiogram revealed consolidation in the left lower lobe, bilateral pulmonary
emboli and a small left sided pneumothorax not visible on the admission Chest X-ray.

Which of her clinical problems requires the most urgent attention?

Chest infection
B

COPD exacerbation
C

Pneumothorax
D

Pulmonary Embolism
E

Type 2 Respiratory Failure

This patient has a CURB65 score of 1 hence the chest sepsis is mild. Her pneumothorax is small and would be high risk to treat
with aspiration and should resolve spontaneously. Her ventilatory failure is mild and likely secondary to chest infection and
pulmonary emboli, thus should respond to treatment of these drivers. The use of Non-Invasive Ventilation would increase the
risk of her developing a tension pneumothorax

Q55.

A 65-year-old man presents to the Emergency Department with acute onset shortness of breath. Arterial blood gases
showed PO2 7.3, PCO2 3.9 and pH 7.45 breathing air. Blood pressure was 130/70 and pulse 120/min, regular. JVP was
elevated 3 cm and there was no ankle oedema. The chest was clear. He had no past history of thromboembolic disease,
but had a positive family history. He had no other known risk factors for venous thromboembolic disease. His CT
pulmonary angiogram was as shown (see image).

Which is the most appropriate management advice?


See image

He should be anticoagulated for life


B

He should be screened for common cancers


C

He should be advised to wear graduated compression stockings for long-haul flights in future
D

He should be treated acutely with unfractionated heparin


E

He should be thrombolysed (assuming no contraindications) in view of the size of the clot

He has suffered a non-massive pulmonary embolus (PE). Massive pulmonary embolus is defined as pulmonary embolus
associated with hypotension (systolic BP <90mmHg) or other adverse features such as cardiac arrest. Thrombolysis is only
currently indicated for massive PE. Life-long prophylaxis after first idiopathic PE should only be considered with the results of
thrombolphilia testing and must be balanced against the risk of bleeding. Life-long prophylaxis remains debatable after first PE,
except in the context of lupus anticoagulant. He should be advised to wear venous compression stockings for future long-haul
flights.

There is an increased risk of cancer being detected within 6–12 months of a first episode of venous thromboembolism (VTE),
particularly in those with no other risk factors and/or recurrent episodes. Previously unrecognised cancer, present in 7–12% of
those with idiopathic VTE, can usually be detected by a combination of careful clinical assessment, routine blood tests, and
chest radiography and, if these are satisfactory, the current consensus is that it is not appropriate to proceed to tests such as
ultrasound, CT scanning or endoscopy.

Q56.

A 35 year old woman was referred to the respiratory outpatient service by her general practitioner who noticed that she was
clubbed and centrally cyanosed. Apart from a cerebellar stroke 10 years ago, which had left no residual deficit, she was
otherwise well. Physical examination was unremarkable, except for the clubbing and cyanosis. A chest radiograph showed a
large, well-defined, rounded lesion in the right lower zone. The following respiratory function test results were obtained: FEV1
93% predicted, FVC 97% predicted, DLCO normal, Oxygen saturation (on air) lying 94% and standing 89%.

Which investigation is most likely to establish the diagnosis?

A
Bronchoscopy
B
CT lungs
C
CT pulmonary angiogram
D
Echocardiography
E
Lung biopsy

This patient has a pulmonary arteriovenous malformation. An upright posture increases the right to left shunt through the
PAVM therefore a drop in oxygen saturations. It also allows microthrombi to reach the arterial circulation and can therefore
occasionally lead to a stroke. A CTPA can demonstrate a feeding pulmonary artery as the cause of the patient’s shunt, and this
can be effectively treated by embolization or surgery (depending on size).

My Masterclass

Create exam

My history

My details
Q57.

A 50 yr old man who had been a heavy smoker since his teens presented to the Emergency Department in respiratory
distress. He had a very sedentary lifestyle, was overweight and known to suffer from severe COPD with a FEV1 of 30%
predicted. His usual medications included inhaled corticosteroid, long acting beta 2 agonist, anti cholinergic and as
required salbutamol, but he did not have a home nebulizer or oxygen. He had been well the day before presentation, but
was woken at night by breathlessness unresponsive to his usual inhalers.

On examination he was distressed with a respiratory rate of 35/min, very poor chest movement and increased resonance.
On auscultation he had globally reduced breath sounds, particularly marked over the left upper chest. A chest X-ray (see
image) showed no consolidation, but very lucent apices with absent lung markings over the left upper chest. ABG
demonstrated pH 7.32, paCO2 7.5 kPa and paO2 6.9 kPa on air.

How would you manage this patient?

See image

CT scan (PA and chest)


B

Insert left sided intercostal chest drain


C

Needle aspiration of left pneumothorax


D

Non-Invasive Ventilation
E

Start treatment for acute exacerbation of COPD


A

Distinguishing bullae from pneumothorax can be very challenging and no aspiration or chest drain insertion should be undertaken if the
pleural margins are not clearly seen. Puncturing a bulla can be fatal to the patient and usually results in prolonged hospitalization. Up to 25%
of acutely exacerbating COPD patients may be suffering from concomitant pulmonary emboli (shown in meta analysis) that may have
triggered the deterioration, so a CTPA and chest should be requested to look for this and to determine whether or not there is a
pneumothorax.

My Masterclass

Create exam

My history

My details

Q58.

A 68 year-old man presented with fever, purulent sputum and disorientation. He was a heavy smoker (30 cigarettes/day for 50 years). Arterial
blood gas analysis results before and two hours after treatment with nebulized bronchodilators, prednisolone and antibiotics were as follows:
FIO2 0.21 0.4 pH 7.32 7.28 PaCO2 (kPa) 7.5 7.39 PaO2 (kPa) 9.1 8.7 HCO3(mmol/l) 25.1 26

What is most important next action in management?

Increase inspired oxygen


B

Initiate non-invasive ventilation


C

Intravenous aminophylline
D

Intravenous corticosteroids
E

Intravenous magnesium

B
Despite treatment with bronchodilators, steroids and antibiotics, the repeat arterial blood gas demonstrates worsening acute respiratory
acidosis (low pH, raised PaCO2, normal HCO3) and he is more hypoxemic. NIV should be the treatment of choice. NIV is successful in 80% of
cases, but fails in 20% in particular those with evidence of pneumonia on the CXR. It is therefore important that the ceilings of therapy be
agreed

Q59.

A 75-year old man presented with anorexia and haemoptysis. He was a smoker of 40 pack year history. Examination revealed signs consistent
with a right pleural effusion. A diagnostic aspiration was performed on the pleural effusion.

Which feature would indicate an exudative pleural effusion?

A
Pleural fluid LDH/serum LDH ratio > 0.5
B
Pleural fluid LDH > two-thirds the upper limit of normal serum LDH
C
Pleural fluid protein/pleural fluid LDH ratio >0.5
D
Pleural fluid protein/serum protein ratio > 0.4
E
Pleural fluid protein of 29 g/L

Light's criteria states that a pleural effusion is exudative when any one of the following conditions is fulfilled

• Pleural fluid LDH/serum LDH ratio > 0.6


• Pleural protein/serum protein ratio > 0.5
• Pleural fluid LDH > two-thirds the upper limit of normal serum LDH

Q60.

An 80-year-old man with COPD was seen in the respiratory outpatient clinic. He was an ex-smoker and had a poor exercise tolerance of
around 100 metres on the flat. His past medical history included localised prostate cancer. He was on treatment with inhaled
umeclidinium/vilanterol (long-acting muscarinic antagonist / long-acting β2-agonist) for his COPD.

He had evidence of significant hyperinflation and gas trapping according to his pulmonary function tests, and a thoracic CT scan revealed
severe upper zone predominant centrilobular emphysema. His ABG on air at rest showed PaO2 8.1 kPa (normal range 11.3–12.6), PaCO2 5.3
kPa (normal range 4.7–6.0) and pH 7.4 (normal range 7.35–7.45).

Which treatment should be considered next?


A
long-term oxygen therapy
B
lung transplantation
C
lung volume reduction surgery
D
nocturnal non-invasive ventilation
E
pulmonary rehabilitation

Pulmonary rehabilitation has been shown to improve dyspnoea, health status and exercise tolerance in stable patients with COPD, as well as
reducing hospitalisation for patients with a recent exacerbation. This patient may be a candidate for lung volume reduction surgery, but he
would need to complete a pulmonary rehabilitation course first. His age and history of malignancy indicates he would not be a candidate for
lung transplantation. His PaO2 is above the cut-off value for consideration of long-term oxygen therapy.

Q61.

A 64-year-old woman presented as an emergency with breathlessness, which had been worsening over the preceding 4 weeks.
She reported that she could now climb a flight of stairs with difficulty, having been able to run in a 10 km fun-run 6 months
earlier. She had been treated for breast cancer at the age of 41 but had no other past medical history and took no regular
medication. She lived with her husband who was in good health. Physical examination revealed stony dullness to percussion and
reduced breath sounds over the right hemithorax. A chest radiograph revealed a large right pleural effusion.

What is the most appropriate initial intervention?

A
diagnostic pleural aspiration of 50 mL fluid under ultrasound guidance, with specimens sent for cytology, microbiology and
biochemical analysis
B
immediate admission to hospital for insertion of a chest drain under ultrasound guidance and talc pleurodesis
C
insertion of a tunnelled intrapleural catheter under ultrasound guidance to permit outpatient pleural drainage
D
non-ultrasound-guided diagnostic pleural aspiration with specimens sent for cytology, microbiology and biochemical analysis
E
therapeutic pleural aspiration of 1,000–1,500 mL fluid under ultrasound guidance, with specimens sent for cytology,
microbiology and biochemical analysis
E

The patient is highly likely to have a malignant pleural effusion. The priorities in her management, in order, are: rapid relief of
current symptoms; rapid diagnosis of the cause of her effusion; an intervention to prevent further effusion-related
breathlessness in future; and the avoidance of unnecessary bed-days in a patient likely to have limited life expectancy. Options
B and C are appropriate for longer-term management of her effusion once a diagnosis is established but do not offer a
diagnosis. Options A and D do not offer symptom relief and it is no longer considered appropriate to carry out pleural
interventions without ultrasound guidance.

The patient is highly likely to have a malignant pleural effusion. The priorities in her management, in order, are: rapid relief of
current symptoms; rapid diagnosis of the cause of her effusion; an intervention to prevent further effusion-related
breathlessness in future; and the avoidance of unnecessary bed-days in a patient likely to have limited life expectancy. Options
B and C are appropriate for longer-term management of her effusion once a diagnosis is established but do not offer a
diagnosis. Options A and D do not offer symptom relief and it is no longer considered appropriate to carry out pleural
interventions without ultrasound guidance.

Q62.

A 69-year-old man was seen in the respiratory outpatient clinic with daytime sleepiness. His past medical history included type 2
diabetes and ischaemic heart disease. His body mass index (BMI) was 29 and his score on the Epworth Sleepiness Scale was 18
(elevated). He had an overnight sleep study that showed an apnoea–hypopnoea index of 35 (elevated). Mean oxygen saturation
was 92% (normal range 94–98%) and a waking capillary blood gas showed PCO2 5.5 kPa (normal range 4.7–6.0), pH 7.39
(normal range 7.35–7.45).

Which initial therapy should be recommended?

A
mandibular advancement device
B
overnight continuous positive airway pressure (CPAP)
C
overnight non-invasive ventilation (NIV)
D
uvulopalatopharyngoplasty
E
weight loss surgery

B
This patient has evidence of severe obstructive sleep apnoea (OSA) syndrome as evidenced by his markedly elevated apnoea–
hypopnoea index in combination with a high score on the Epworth Sleepiness Scale and elevated body mass index (BMI). The
normal waking capillary blood gas and mean oxygen saturation of 92% (normal range 94–98%) indicates the predominant
picture is of OSA rather than of hypoventilation. Nocturnal CPAP is the most appropriate therapy in this circumstance. A
mandibular advancement device is unlikely to be successful in this situation of severe OSA. Weight loss surgery could also be
considered, but CPAP should be started in the first instance.

Q63.

A 25-year-old man was admitted with a history of haemoptysis, cough and dyspnoea. He had been previously fit, but had
smoked 20 cigarettes a day for the last 5 years. Urine dipstick showed haematuria and proteinuria. A full blood count revealed a
microcytic, hypochromic anaemia. A chest radiograph showed diffuse alveolar infiltrates. Lung function tests found normal
spirometry and a TLCO of 135% predicted. What is the most likely diagnosis?

A
Community acquired pneumonia
B
Congestive cardiac failure
C
Goodpasture's syndrome
D
Pulmonary emboli
E
Pulmonary tuberculosis

Goodpasture's syndrome is characterised by diffuse alveolar haemorrhage and glomerulitis. Men are commonly affected with
most cases occurring between the ages of 20-30 years. It is more likely to occur in smokers. Exposure to smoke and volatile
hydrocarbons is associated with initiation as well as exacerbation of the disease. The anti glomerular basement antibody is
present in up to 90% of the patients. Renal histology usually shows a focal segmental necrotizing glomerulitis with crescent
formation.

The TLCO is increased during active bleeding and can be used to monitor the disease. An increase above 30% of baseline is
highly suggestive of an intra-alveolar bleed.

Q64.
A 47-year-old man was admitted with community-acquired pneumonia. His chest X-ray showed right middle lobe consolidation.
Blood cultures were positive for Streptococcus pneumoniae and his urinary pneumococcal antigen was positive. He was treated
with intravenous benzylpenicillin. On day 3 of treatment his C-reactive protein (CRP) had not fallen and he continued to be
pyrexial.

What is the most appropriate course of action?

A
change antibiotics to piperacillin/tazobactam
B
perform a lumbar puncture
C
request a chest X-ray
D
request a CT pulmonary angiogram
E
request an echocardiogram

High-level penicillin resistance is rare in Streptococcus pneumoniae. The failure to improve with appropriate antibiotic therapy
suggests that the patient may have developed a complication of his pneumonia. The most likely cause is a parapneumonic
effusion, which may have become infected. The most appropriate investigation is therefore a chest X-ray. If this confirmed the
presence of a pleural effusion he would then require an ultrasound-guided pleural aspiration.

My Masterclass

Create exam

My history

My details

Q65.

The chest CT is shown from a woman of 50 who underwent lumpectomy, followed by radiotherapy to her right breast and axilla
for breast carcinoma. She presents with cough and breathlessness. What is the likely diagnosis
See image

Allergic bronchopulmonary aspergillosis (ABPA)

Bacterial pneumonia.

Eosinophilic pneumonia

Radiation pneumonitis

Recurrent breast cancer

The CT shows sharply demarcated consolidation in the right upper lobe in a region within the likely field of radiotherapy that
would encompass the breast and axilla. The sharp border to the area of abnormality is characteristic of radiation pneumonitis.
Radiation can produce organising pneumonia or interstitial fibrosis within the lung.
ABPA is characterized by proximal bronchiectasis.

Eosinophilic pneumonia produces patchy peripheral pulmonary infiltrates and is characteristically very responsive to steroid
treatment.

Recurrent breast metastases are unlikely in view of the air bronchograms within the abnormal area.

Segmental pneumonia is a possible diagnosis but less likely than radiation pneumonitis.

Q66.

A 47-year-old building labourer, a smoker of 25 pack-years, was referred to the respiratory outpatient service with a cough and
breathlessness. He had noticed a little loss of weight. His past medical history was notable for a myocardial infarction 3 years
ago, from which he had made a good recovery such that he remained able to work.

On physical examination, he had clubbing of the fingernails, but there were no other abnormal signs. A CT scan of the chest and
upper abdomen revealed a 2.3 cm right upper lobe nodule and enlarged (1.5 cm short axis) lymph nodes in the right hilar and
right paratracheal stations, but no other abnormalities. His lung function tests, full blood count, urea and electrolytes (U/E), liver
function tests (LFTs) and calcium were normal

Which is the most appropriate next investigation?

A
a cardiopulmonary exercise test
B
a CT-guided needle biopsy of the right upper lobe nodule
C
a flexible bronchoscopy
D
a positron emission tomography (PET)-CT scan
E
an endobronchial ultrasound and mediastinal lymph node biopsy

The priority is to establish diagnosis and staging as rapidly as possible and with the minimum number of invasive tests. This man
has potentially operable lung cancer. A PET-CT scan will identify extrathoracic metastasis in approximately 10% of patients and
should be performed in all radically treatable patients with lung cancer. By performing the PET-CT scan first, a biopsy of the
highest-stage disease can be performed, thus minimising invasive testing.

Q67.
A 70 year old heavy smoker, but with no significant past medical history, presented to the Emergency Department acutely short
of breath with oxygen saturations of 80% on air, pulse 120/min and BP 150/95 mmHg. He was started on high flow oxygen via a
non-rebreathe mask, but his saturations did not come above 90%. He was therefore switched to CPAP and immediately began
to improve, with saturations rising to 98% on 35% oxygen. His pre-CPAP chest radiograph is shown (see image).

What is the most likely diagnosis?

See image
A

Cryptogenic organising pneumonia


B

Pneumocystis jirovecii pneumonia


C

Pulmonary oedema
D

Streptococcal pneumonia
E

Usual interstitial pneumonia

C
This CXR shows bilateral interstitial shadowing in a ‘bats wing’ pattern, characteristic of pulmonary oedema. CPAP can improve
this significantly by increasing intra-alveolar pressure and driving fluid back into the circulation, recruiting more alveoli and
therefore improving oxygenation.

Q68.

A 62 year old woman with a history of diabetes and ischaemic cardiomyopathy presented to the Emergency Department with
breathlessness. She had never smoked.
o
On examination she was overweight (BMI 29), with temperature 37.4 C, pulse 80/min, respiratory rate 14/min, BP 136/88
mmHg. Abnormal findings included JVP +5cm, a few bi-basal crackles and expiratory wheeze, and pitting oedema to mid shins.

Blood test results included a normal full blood count, normal troponin I, and mildly elevated CRP (12). Arterial blood gas (on air)
showed pH = 7.349, pO2 = 9.9kPa, pCO2 = 7.10kPa, HCO3 = 33. A chest X-ray revealed mild pulmonary oedema.

What is the most likely cause of the hypercapnoea?

A
Asthma
B
Chest infection
C
Congestive cardiac failure
D
Obstructive sleep apnoea
E
Pulmonary embolism

CCF rarely causes significant hypercapnoea and respiratory acidosis unless very severe and chronic. Asthma or chest infection do
not cause hypercapnoea except in the peri-arrest setting. Up to 50% of patients with CCF can develop OSA due to altered lung
mechanics. The fact that she is borderline obese further increases risk of OSA

Q69.

A 56 year old male lorry driver presented to clinic with a 6 month history of increasing shortness of breath. He had a past
medical history of diet controlled type II diabetes, hypertension and hypercholesterolaemia. He was a non-smoker. He was
obese (BMI 39.9 kg/m2), with normal cardiac and respiratory findings on examination. A CXR was reported as showing clear lung
fields. Pulmonary function tests showed an FEV1 of 1.2 litres (34% predicted), FVC of 1.6 (38% predicted) and a carbon
monoxide transfer factor of 40% predicted, which corrected for alveolar volume to 114% predicted.

What management should be recommended?

A
ACE inhibitor
B
Long acting muscarinic antagonist inhaler
C
Short acting beta-2 agonist inhaler
D
Pirfenidone
E
Weight loss

The restrictive spirometry in combination with the reduced transfer factor correcting for alveolar volume is suggestive of an
extra-thoracic restriction. History and examination should focus on signs of obesity hypoventilation, such as early morning
headaches and signs of right heart failure. Additional questions with regards to daytime somnolence should also be asked.

Q70.

A 65 year old man had become progressively more breathless over the last two years. He had smoked 20 cigarettes a day for the
last 50 years, but reduced to 12 a day over the last 6 months. On examination he had an overinflated chest, some early
inspiratory crackles and an expiratory wheeze. He had mild ankle oedema. Investigations showed that his FEV1 was 30% of
predicted, with a forced expiratory ratio (FEV1/FVC) of 40%. His oxygen saturation at rest breathing room air was 90%.

Which treatment would improve his life expectancy?

A
Inhaled long acting bronchodilators
B
Oxygen administered overnight during sleep
C
Pneumococcal vaccination
D
Pulmonary rehabilitation
E
Smoking cessation
E

Smoking cessation has been found in a number of studies to slow the rate of decline of lung function, and prognosis is linked to
percent predicted FEV1. Long term oxygen is likely to be indicated for this patient with 90%oxygen saturation at rest breathing
room air, if he agrees to stop smoking. The criteria for long term oxygen therapy in the UK are a PaO2 <7.3kPa or <8kPa with
signs of right heart failure (e.g. ankle oedema). Overnight oxygen is insufficient to improve median survival: use should be
greater than 15 hours daily, and as close to 24 hours as possible.

My Masterclass

Create exam

My history

My details

Q71.

A 24 yr old man of afro-Caribbean descent had recently returned to the UK after the spending six months working as a volunteer
nurse in Sierra Leone. He presented with a two month history of fatigue, weight loss, night sweats, and a dry cough. He had
some occasional long standing joint discomfort, but no dryness of his eyes or mouth.

On examination he had erythema nodosum on his lower limbs and a BCG scar. His chest X-ray showed bilateral hilar adenopathy
(see image). Other test results revealed mild anaemia (Hb 118 g/L), normal liver blood tests, a negative HIV test, normal serum
ACE, and normal urinary calcium excretion. His sputum microscopy revealed no AAFB's. Endobronchial ultrasound guided
biopsies of his hilar nodes showed non-caseating granulomas, with no AAFB's seen. Culture results were awaited.

What is the best management plan?


See image

No specific treatment until culture results available


B

Refer for thoracoscopic lymph node biopsy


C

Start anti-mycobacterial therapy


D

Start anti-mycobacterial therapy and oral steroids


E

Start oral steroids

Although this man is a typical sarcoid patient, his clinical presentation is atypical and biopsies of TB lymph nodes often do not
contain caseation. His chest X-ray changes are asymmetrical, and Sierra Leone has 600 cases per 100 000 population of TB,
putting him at significant risk. Until negative culture results are received he should be managed as a case of MTB. Sarcoid is a
diagnosis of exclusion and no therapy should be initiated until other causes are ruled out.
Q72.

A 37 year man presented with a cough, fever and shortness of breath. On examination there were a few crackles in both lungs,
more marked on the left. A full blood count showed an elevated white cell count and low haemoglobin. A CXR showed a patchy
left sided infiltrate.

Which further diagnostic test would be most appropriate?

A
Bronchoscopy for bronchoalveolar lavage and culture
B
Cold agglutinins
C
High Resolution CT scan of the chest
D
Urinary legionella antigen
E
Urinary pneumococcal antigen

Patients with mycoplasma pneumonia may present with a haemolytic aenamia in up to 60% of cases. This is due to IgM
antibodies to the I antigen of red blood cell membranes.

Q73.

A 60-year-old man was referred to the medical assessment unit from the Emergency Department with a two-day history of
coughing up sputum and blood. A chest X-ray revealed no abnormalities.

What should be the next management step?

A
Admit for inpatient bronchoscopy
B
CT chest
C
D-dimer
D
Refer for outpatient bronchoscopy
E
Start oral tranexamic acid

Patients with haemoptysis and a normal chest radiograph can be managed as an outpatient by the respiratory team. Patients
with haemoptysis and an abnormal chest radiograph might also be managed as an outpatient, but should have an urgent (within
2 weeks) CT chest. Tranexamic acid should only be considered if pulmonary embolism has been ruled out.

My Masterclass

Create exam

My history

My details

Q74.

A 27-year-old man was admitted to the respiratory ward with cough, fever and purulent sputum production. In the weeks prior
to his admission he had suffered from a sore throat and had difficulty swallowing. He had no relevant past medical history and
took no regular medication. On examination the right side of his neck was tender, and ultrasound with Doppler revealed a right
internal jugular vein thrombus. His chest X-ray showed several cavitating lesions in both lung fields.

What is the most likely responsible organism?

Fusobacterium necrophorum
B

Klebsiella pneumoniae
C

Mycobacterium tuberculosis
D

Staphylococcus aureus
E

Streptococcus pneumoniae
A

Fusobacterium necrophorum causes Lemierre’s syndrome. This is a pharyngeal infection most commonly occurring in young
adults. The infection spreads through the soft tissues of the neck and carotid sheath causing jugular vein thrombosis.
Embolisation to the lung can occur, causing cavitation and abscess formation.

Q75.

A 26-year-old man with known cystic fibrosis presented with weight loss, urinary frequency and frequent infective exacerbation
of his chest problem. The most likely complication to explain this is:

A
Amyloidosis
B
Cirrhosis
C
Diabetes mellitus
D
Diverticulitis
E
Renal tubular acidosis

Approximately 20% of adult patients with cystic fibrosis develop diabetes mellitus. The reason for this is direct damage to the
insulin-producing islets. The commonest manifestations of diabetes in CF patients are weight loss, increased frequency of
infective exacerbation, as well as decline of lung function tests. Increased urinary frequency is not specific for diabetes in adult
CF patients.

Q76.
A 55 year old White British man with a history of alcohol abuse presented to hospital with a two week history of cough and right
sided pleuritic chest pain. Vital signs included temperature 38.1oC, pulse 110/min, BP 130/75 mmHg, and Sats 97% (on air).
Examination of the chest revealed a dull percussion note and decreased air entry at the right base.

Chest x-ray confirmed a moderate right sided pleural effusion, and a diagnostic pleural tap was performed by the admitting
team under direct ultrasound guidance.

Pleural fluid results were as follows: Microscopy - Gram stain negative (culture results awaited)
Protein - 30g/L
Glucose - 2.0 mmol/L pH - 7.1

What is the most appropriate treatment?

A
Antibiotics for community acquired pneumonia (intravenous)
B
Antibiotics for community acquired pneumonia (intravenous) and for tuberculosis (oral)
C
Antibiotics for community acquired pneumonia (intravenous) and intercostal chest drain
D
Antibiotics for community acquired pneumonia (oral)
E
Antibiotics for community acquired pneumonia (oral) and for tuberculosis (oral)

This man has an empyema which requires prompt treatment with IV antibiotics and drainage. Thoracic surgical input may be
required if it not resolve. Empyema is more common in the immunocopromissed and alcoholics. A pleural effusion with a pH of
less than 7.2 requires a chest drain as per British Thoracic Society guidelines.

Q77.

A 50-year-old man presents with breathlessness that has got gradually worse over three months. He has long-standing atrial
fibrillation, for which he takes warfarin and amiodarone. On examination his pulse is 80/min in AF and he has fine bibasal
crackles, but there are no other abnormal physical signs. The most likely diagnosis is:

A
amiodarone-induced interstitial lung disease
B
congestive cardiac failure
C
idiopathic interstitial lung disease
D
pulmonary embolism
E
pulmonary haemorrhage.

Congestive cardiac failure would clearly need to be considered in a man with known cardiac disease, evidenced in this case by
atrial fibrillation, but this is not a sustainable diagnosis in the absence of signs other than basal crackles. However, you would
obviously look carefully for raised JVP, displaced apex, LV or RV heaves, added heart sounds and peripheral oedema before
excluding this diagnosis.

Chronic pulmonary embolism could present with insidious breathlessness but is not likely in a man who is on warfarin, and there
are no signs to support the diagnosis (raised JVP, RV heave, loud P2, RV gallop).

The diagnosis of idiopathic interstitial lung disease cannot be made in the presence of a known secondary cause of interstitial
lung disease.

Q78.

A 20-year-old woman presented to the Emergency Department with acute breathlessness and wheeze on a background of
asthma. Vital signs included pulse 110/min, BP 145/87 mmHg, respiratory rate 22/min, and SpO2 100% (on FiO2 100%). A chest
radiograph was normal and an arterial blood gas (on FiO2 100%) showed PO2 20kPa, PCO2 5.6kP, pH 7.4. She had been given
steroids and nebulisers.

What is the correct management?

A
Aminophylline (intravenous)
B
Magnesium sulphate (intravenous)
C
Repeat salbutamol nebuliser
D
Transfer to a respiratory ward
E
Urgent referral to ICU

Acute asthma typically causes a respiratory alkalosis in view of the associated tachypnoea. A normal PCO2 suggests that the
patient is tiring and that she needs to be assessed urgently by ICU for consideration of intubation and ventilation.

My Masterclass

Create exam

My history

My details

Q79.

A 54-year-old smoker with chronic obstructive pulmonary disease was referred for advice regarding further management. His
only medication was a salbutamol inhaler that he used on average 4 times a day. His spirometry was as follows: FEV1 1.7 litres
(64% predicted), FVC 2.40 litres (75% predicted), and FEV1/VC 56%. Post bronchodilator FEV1 was 2.5 litres (92% predicted).
What would you prescribe?

Corticosteroid (inhaled, regularly)


B

Long acting anti-cholinergic (inhaled, regularly)


C

Long acting beta2 agonist (inhaled, regularly)


D

Methyloxanthine (oral, regularly)


E

Salbutamol (inhaled, regularly)

A
A significant bronchial reversibility of 44% with normal postbronchodilator FEV1 is consistent with asthma not COPD. Asthma
guidelines should be therefore followed and regular inhaled corticosteroids prescribed.

My Masterclass

Create exam

My history

My details

Q80.

A 71-year-old man with idiopathic pulmonary fibrosis presented to the Emergency Department with breathlessness due to a
first episode of a spontaneous left pneumothorax. His respiratory rate was 16 / min, with SaO2 97% on room air. The chest
radiograph showed a left-sided pneumothorax with the lung edge 4 cm from the edge of the thoracic cavity. What is the most
appropriate management?

Aspirate the chest


B

Insert an intercostal drain


C

Observe
D

Refer for medical pleurodesis


E

Refer for surgical pleurectomy

This is a secondary spontaneous pneumothorax, in which case drainage is the most appropriate form of management. The only
indication to aspirate would be a patient who was not breathless, aged less than 50 and a smaller pneumothorax (<2cm) (BTS
guidelines 2003). Therefore, as a general rule, these types of pneumothorax require drainage. There is no indication to refer to
the surgeons unless the pneumothorax does not resolve with tube drainage or if this is a repeat episode.

Q81.
A 24-year-old woman complains of excessive daytime sleepiness, and also mentions that she has fallen asleep at work. Her
partner says that he has sometimes found her asleep during supper. A diagnosis of narcolepsy is made. Which of the following is
true?

A
Cataplexy is pathognomic.
B
Narcoleptic patients have normal nocturnal sleep pattern.
C
Nasal Continuous Positive Airway Pressure (CPAP) is the treatment of choice.
D
Nocturnal apnoea spells are characteristic of this condition.
E
Patients with narcolepsy have raised hypocretin levels in the cerebrospinal fluid (CSF).

Narcolepsy is characterised by cataplexy, sleep paralysis, hypnagogic hallucinations and excessive daytime sleepiness. Over 24
hours, patients with narcolepsy do not sleep more than normal controls, but they are prone to fall asleep throughout the day,
often at inappropriate times. Nocturnal apnoea spells are not a feature. Such patients lack hypocretin in the CSF. In patients
with narcolepsy, night sleep is often interrupted by repeated awakenings and terrifying dreams.

Q82.

A 60-year-old man presented at 22.00h to the Emergency Department with a 6 week history of worsening breathlessness and
left sided pleuritic chest pain, his breathlessness having deteriorated significantly over the prevoius 4 days. He had a dry cough
but denied fevers. His past medical history included hypertension, diabetes and chronic kidney disease (Stage 3), and he was a
lifelong heavy smoker.
o
On examination he was breathless at rest using his accessory muscles. His vital signs were temperature 37.4 C, pulse 120/min,
BP 90/50 mmHg, and respiratory rate 32/min, with SaO2 89% on 60% oxygen. His heart sounds were normal and he had
reduced air entry with stony dullness throughout the left hemithorax. His ECG showed sinus tachycardia and his CXR confirmed
a massive left pleural effusion without mediastinal shift. The following blood tests were obtained: Hb=11.4 g/dL, WBC=12.1
×10&sup9;/L, PLT=250 ×10&sup9;/L, CRP=40, INR=1.2

What is the most appropriate immediate management?

A
Insert chest drain immediately (without requesting ultrasound guidance)
B
Make urgent referral to respiratory services for chest drain insertion the following morning
C
Perform diagnostic pleural tap immediately
D
Request urgent chest ultrasound to mark site for chest drain insertion immediately afterwards
E
Request urgent CT scan of chest, abdomen and pelvis

This patient presented with massive pleural effusion causing hemodynamic compromise and requiring high FiO2 to maintain
oxygen saturation. Even though he presented out-of-hours, his clinical situation requires an urgent chest drain. The BTS
guidelines 'strongly recommend that all chest drains for fluid should be inserted under image guidance'. The technique of
marking the chest for subsequent drain insertion is acceptable for large effusions and is the preferred option of those listed in
this case

Q83.

A 66-year-old man with known chronic obstructive pulmonary disease (COPD) presents via the emergency department with an
acute exacerbation. The ambulance crew have put him on high flow oxygen using a reservoir mask. His initial arterial blood
gases are: pH 7.32, pCO2 7.1 kPa, pO2 15.2kPa, standard base excess -1.7mmol/l, bicarbonate 32mmol/l. The patient is alert and
orientated. Appropriate initial management would be:

A
Application of non-invasive ventilation via a tight-fitting facemask
B
Immediate rapid sequence induction, intubation and ventilation
C
Intravenous administration of a bolus of aminophylline (250mg) followed by a maintenance infusion
D
Reduction in inspired oxygen concentration to 28% via a Venturi mask and recheck arterial blood gases in 20 minutes
E
Reduction in inspired oxygen concentration to 60% via a Venturi mask and recheck arterial blood gases in 20 minutes

E
The patient has evidence of type II respiratory failure with elevation of the pCO2. Fortunately he is alert and orientated which
allows you some time to initiate treatment and try to avoid intubation and ventilation (once intubated it often proves extremely
difficult to wean such patients from the ventilator). Part of the problem may be related to administration of high flow oxygen in
someone who normally relies on hypoxic drive to stimulate respiration. The ambulance crew may well have been right to give
high flow O2 initially as hypoxia kills far quicker than hypercarbia: the oxygen concentration should now be reduced slowly
whilst rechecking blood gases regularly. Switching abruptly to low concentration oxygen may render the patient both hypoxic
and hypercarbic (a bad combination).
IV aminophylline is part of the armament of treatment in such cases but other treatments should have been tried first. Similarly,
non-invasive ventilation may be extremely useful and avoid intubation. It is increasingly used outside the ICU setting.

My Masterclass

Create exam

My history

My details

Q84.

A 28 year old woman presented to the Emergency Department with left sided chest pain. This had started two days previously
when she was exercising, and it was getting worse. It was dull in nature, did not radiate, increased with chest movements, and
was not associated with any other symptoms. She had no significant past medical history. She was not on any regular
medications, apart from the oral contraceptive pill.

On examination her respiratory rate was 15 /min and oxygen saturation 98% (breathing air).

Investigations:

D dimer positive
haemoglobin 110 g/L (130–180)
white cell count 9.5 109 /L (4.0–11.0)
platelet count 250 109/L (150–400)
erythrocyte sedimentation rate 10 mm/1st h (<20)
serum sodium 138 mmol/L (137–144)
serum potassium 3.9 mmol/L (3.5–4.9)
serum urea 5.3 mmol/L (2.5–7.0)
serum creatinine 90 μmol/L (60–110)

arterial blood gases breathing air:


PaO2 12.8 kPa (>10 kPa on air)
pH 7.40 (7.35 – 7.45)
PaCO2 5.6 kPa (4.7 – 6.0 kPa)

ECG - sinus rhythm, rate 80 beat per minute, no ischaemic changes.

What is the most likely diagnosis?

A
Oesophagitis
B

Musculoskeletal chest pain


C

Pneumonia
D

Pneumothorax
E

Pulmonary embolism

The clinical history fits best with musculoskeletal chest pain. The D dimer is most likely to be a false positive here, but given that
it is elevated, the prudent approach would be to arrange imaging to exclude PE.

Q85.

A 72-year-old man presented to the emergency department with a 2-day history of fever and cough. His past medical history
included hypertension and gastro-oesophageal reflux. On examination he was alert and orientated. His vital signs included
temperature 38.7°C, pulse 120 beats per minute, blood pressure (BP) 75/40 mm/Hg and respiratory rate 30 breaths per minute.
There were bronchial breath sounds at the right base. His blood tests included C-reactive protein (CRP) 280 mg/L (normal
threshold <10 mg/L), white cell count (WCC) 18 × 109/L (normal range 4–11), urea 17 mmol/L (normal range 2.5–7.0), creatinine
220 μmol/L (normal range 60–110). His chest X-ray showed right lower lobe consolidation.

What is his CURB-65 score?

A
1
B
2
C
3
D
4
E
5

D
The CURB-65 score is used to assess the severity of community-acquired pneumonia. A point is scored if each of the following is
present: confusion (Abbreviated Mental Test score ?8), urea >7 mmol/L, respiratory rate ?30 breaths per minute, systolic blood
pressure (BP) <90 mmHg and/or diastolic BP ?60 mmHg, age ?65. This patient therefore scores 4, which gives him a predicted
mortality of 28% at 30 days, signifying severe pneumonia.

My Masterclass

Create exam

My history

My details

Q86.

A 43-year-old woman was admitted with infective exacerbation of asthma. She responded to treatment but the house officer
was concerned that Aspergillus fumigatuswas cultured from her sputum. Subsequently arranged serum total, IgE level was
elevated at 438 ng/ml (normal 40-180 ng/ml), RAST to Aspergillus fumigatus was class III, Aspergillus fumigatus precipitins were
negative. What would be the most appropriate action?

Chest X ray

High dose oral corticosteroids

High dose oral corticosteroids and Itraconazole

Itraconazole

No change in medication.

With negative Aspergillus fumigatus precipitins and serum total IgE less than 1000 ng/ml, this patient is unlikely to have Allergic
Bronchopulmonary Aspergillosis (ABPA).
Many asthmatics have one or more findings of ABPA but do not meet full criteria for the diagnosis. The presence of isolated
ABPA features may cause diagnostic confusion but does not appear to affect prognosis. Features of ABPA which are found
commonly in asthmatics without ABPA include:

 Positive immediate skin reactivity to Aspergillus fumigatus, which is present in 20-30% of all asthmatics
 Positive serum precipitins to Aspergillus, which occur in 10% of asthmatics without ABPA and in 10% of non-asthmatic
patients with chronic lung disease
 Recurrent mucoid impaction and atelectasis
 Peripheral blood eosinophilia and elevation of serum total IgE.

My Masterclass

Create exam

My history

My details

Q87.

A 65 year old man with advanced chronic obstructive pulmonary disease (FEV1 29% predicted) presented with to the Emergency
Department with a nearly complete right spontaneous pneumothorax. A 12F Seldinger intercostal drain was inserted, and a
post-procedural chest X-ray revealed the drain to be well placed. The patient was comfortable and was transferred to a medical
ward.

Some hours later the patient became very distressed. On examination the drain was ‘swinging’ and bubbling intermittently, and
there was significant surgical emphysema on the right side. Other significant findings were labored breathing with use of
accessory muscles, central cyanosis, that the arterial pulse at the wrist could be felt intermittently, the JVP was elevated, and
that there were reduced breath sounds in the chest. Arterial blood gas analysis revealed hypoxia, acidosis and a rise in pCO2
from 5.6 kPa (on admission) to 9.7 kPa.

What is the most likely diagnosis?

Acute coronary syndrome


B

Cardiac tamponade
C

Left pneumothorax
D

Pneumonia
E

Right tension pneumothorax


E

It would be very unlikely to develop a second contralateral pneumothorax, and sepsis would take more time to develop. A large
air leak can overwhelm a small chest drain and lead to tension, and thus one should never be reassured by the presence of a
drain if features of tension are present.

Q88.

A 50-year-old Indian male accountant gives a history of 3-months cough, weight loss and haemoptysis. He was found to
have some nasal crusting but otherwise examination was normal. His CRP and ESR are elevated but he has normal
biochemistry except for an elevated creatinine of 400. His chest radiograph showed a few nodules. A CT thorax is shown
(see image). What is the most likely diagnosis?

See image

Bronchial carcinoma

Staphylococcal pneumonia

Systemic lupus erythematosus (SLE)

Tuberculosis

E
Wegener’s granulomatosis

The two most appropriate diagnoses were Wegener’s and tuberculosis. However, given the renal dysfunction and nasal disease,
a vasculitis is most likely. The CT shows multiple and occasionally cavitating nodules. Although staphylococcal infection
cavitates, the history is too long for this. Appropriate investigations would involve bronchoscopy and serum ANCA.

Q89.

A 67-year-old man has been diagnosed with chronic obstructive pulmonary disease (COPD). Spirometry confirms severe
COPD with a FEV1 of less than 30% predicted. In the last 12 months he has been admitted to hospital on four occasions
with COPD exacerbation. Which one of the following statements is true?

A
Exacerbations can be reduced by daily oral steroids.
B
Long-acting anticholinergics can reduce exacerbation rate.
C
Long-acting beta 2 agonists reduce exacerbations.
D
Nebulisers help to reduce exacerbations.
E
Oral theophyllines can reduce rate of exacerbations.

Severe COPD is diagnosed if the FEV1 is less or equal to 30% predicted. Studies have shown that patients treated with long
acting anticholinergic (tiotropium) had fewer exacerbations per year. Use of rescue medication was often less in patients on
tiotropium.

Q90.

A 60-year-old gentleman is referred to the hospital as a routine chest radiograph has shown bilateral apical scars with possibility
of cavity formation. He is complaining of some cough for over 8 months and is known to suffer from mild chronic obstructive
pulmonary disease (COPD). There is no history of immunosuppression and repeated sputum samples have
grown Mycobacterium avium complex. Which of the following is true?

A
Extrapulmonary disease is uncommon in immunocompetent patients.
B
It commonly affects young fit men.
C
Patient should be notified within 1 week.
D
Surgery has no role in treatment.
E
Treatment should be the same as Mycobacterium tuberculosis but with an 8-month continuation phase using isoniazid and
rifampicin.

M. avium complex (MAC) is a saprophyte and can be found in soil, water, dust etc. It typically affects middle age and old men
with underlying lung diseases such as COPD, old TB, bronchiectasis. It may be contaminant, hence to make a diagnosis there
should be radiographic evidence of disease (cavities etc), and it should be isolated in at least three sputum samples.

Extrapulmonary disease is seen in immunosupressed patients. Recommended treatment is with rifampicin and ethambutol for
24 months.

In fit patients, surgery resection of the diseased lung along with chemotherapy can cure.

Q91.

A 64-year-old woman with newly-diagnosed small cell carcinoma discusses her further treatment options with the respiratory
consultant, the cardiothoracic surgeon and the local oncologist. Which of the following statements are correct?

A
Brain metastases are uncommon in patients with small cell lung cancer.
B
Management of small cell carcinoma should include combined chemotherapy/radiotherapy.
C
Patients with small cell lung cancer always benefit from surgery.
D
Radiotherapy has no role to play in the treatment of small cell lung cancer.
E
Small cell carcinoma is rarely responsive to chemotherapy.
B

Because small cell lung cancer has a propensity to spread early, surgical resection is not usually considered as part of the routine
treatment. Patients diagnosed by biopsy are treated by combined radiotherapy and chemotherapy. Concurrent radiotherapy
and chemotherapy provides better 5-year survival rates than sequential therapy. Patients are at risk of brain metastases and
one study suggested a 50% incidence 2 years after diagnosis.

Q92.

A 75-year-old woman, recently returned from a holiday abroad, presented with 10 day history of right sided pleuritic chest pain
and breathlessness. She had a dry cough but denied any fevers. Her past medical history included ischaemic heart disease,
hypertension, diabetes and metastatic bowel cancer.

On examination her vital signs were temperature 36.9°C, pulse 95/min, BP 140/75 mmHg, and respiratory rate 20/min, with
SaO2 96% on air. Her heart sounds were normal and the chest was clear. Her ECG showed sinus rhythm and the CXR was
normal. The following blood tests were obtained: WBC=11.1 ×10&sup9;/L, Hb=11.5 g/dL, Plt=220 ×10&sup9;/L, CRP=20, D-
dimer=negative

What is the best next investigation?

A
Atypical serology
B
CT pulmonary angiogram
C
Echocardiogram
D
Lung function tests
E
Re-staging CT thorax /abdomen / pelvis

This patient is at moderate/high risk pulmonary embolism and according to guidelines a negative D-dimer is not enough to
exclude PE. She should be investigated further and from the provided list, CTPA is the best next investigation.

Q93.
A 73-year-old man with known chronic obstructive pulmonary disease (COPD) is admitted with symptoms of worsening
breathlessness and confusion. His arterial blood gases are as follows: pH 7.20, PO2 6.0 kPa (45 mmHg), PCO2 9.1 kPa (68
mmHg). Which one of the following statements regarding his management is correct?

A
Emphysema is a contraindication for nasal ventilation because of the risk of pneumothorax
B
Low arterial pH in COPD is associated with a high mortality rate
C
Mechanical ventilation with tracheal intubation should be considered
D
Non-invasive positive pressure ventilation (NIPPV) is not appropriate for patients with COPD
E
Patients with COPD are particularly difficult to wean from mechanical ventilation

This patient should be considered for mechanical ventilation. Non-invasive intermittent positive pressure ventilation (NIPPV) is a
useful method of treatment of acute or chronic respiratory failure. The results are best in patients with type II respiratory failure
without acidosis. NIPPV should be either a first step of management in those who are suitable for mechanical ventilation or the
management of choice in patients who are deemed to be unsuitable for intubation and ventilation. The rate of pneumothorax is
small, but is higher in machines using volume-cycled mode rather than pressure cycled mode.

The rate of successful weaning from mechanical ventilation of appropriately selected patients with COPD is similar to that for
other diseases.

Q94.

A 62-year-old man presented with increasing shortness of breath. He was a current smoker with over 20 packs per year
exposure. He was found to have a right-sided pleural disease. A final diagnosis of mesothelioma was made. Which of the
following statements is correct?

A
Chemotherapy is the most appropriate treatment.
B
Patients should be advised about state compensation and entitlement to a disability pension.
C
Pleural aspiration is usually adequate for a diagnosis.
D
Radiotherapy is the primary treatment.
E
Sarcomatous mesothelioma has a good prognosis.

The correct answer is B. As asbestos is associated with the development of mesothelioma, all patients should be advised that
they may be eligible for sate compensation and disability benefits. These claims must be made within 3 years of initial diagnosis.

Q95.

An 80-year-old man with COPD was seen in the respiratory outpatient clinic. He was an ex-smoker and had a poor
exercise tolerance of around 100 metres on the flat. His past medical history included localised prostate cancer. He was
on treatment with inhaled umeclidinium/vilanterol (long-acting muscarinic antagonist / long-acting β2-agonist) for his
COPD.

He had evidence of significant hyperinflation and gas trapping according to his pulmonary function tests, and a thoracic CT
scan revealed severe upper zone predominant centrilobular emphysema. His ABG on air at rest showed PaO2 8.1 kPa
(normal range 11.3–12.6), PaCO2 5.3 kPa (normal range 4.7–6.0) and pH 7.4 (normal range 7.35–7.45).

Which treatment should be considered next?

A
long-term oxygen therapy
B
lung transplantation
C
lung volume reduction surgery
D
nocturnal non-invasive ventilation
E
pulmonary rehabilitation

E
Pulmonary rehabilitation has been shown to improve dyspnoea, health status and exercise tolerance in stable patients with
COPD, as well as reducing hospitalisation for patients with a recent exacerbation. This patient may be a candidate for lung
volume reduction surgery, but he would need to complete a pulmonary rehabilitation course first. His age and history of
malignancy indicates he would not be a candidate for lung transplantation. His PaO2 is above the cut-off value for consideration
of long-term oxygen therapy.

Q96.

A 42-year-old woman was admitted to the respiratory ward with a 6-month history of breathlessness, increasing peripheral
oedema, daytime somnolence and waking headache. She had a past medical history of type 2 diabetes, hypothyroidism and
hypertension. On examination her BMI was 33, her chest was clear to auscultation, there was pitting oedema up to her
abdominal wall, and her jugular venous pressure was elevated. She had peripheral capillary oxygen saturation of 89% on air
(normal range 94–98%). Her chest X-ray showed normal lung fields. An ABG test on air showed PO2 7.7 kPa (normal range 11.3–
12.6), PCO2 9.8 kPa (normal range 4.7–6.0), pH 7.36 (normal range 7.35–7.45) and serum bicarbonate 39 mmol/L (normal range
21–29).

Which intervention is most likely to lead to long-term clinical improvement?

A
acetazolamide
B
high-dose diuretic therapy
C
long-term oxygen therapy (LTOT)
D
nocturnal continuous positive airway pressure treatment
E
nocturnal non-invasive ventilation (NIV)

This patient has signs and symptoms of obesity hypoventilation syndrome (OHS) with subsequent right ventricular failure and a
chronic compensated hypercapnia. Nocturnal NIV will treat her overnight hypoventilation and will improve her right ventricular
dysfunction. Diuretic therapy alone will not treat the underlying problem. Continuous positive airway pressure (CPAP) is used to
treat obstructive sleep apnoea (OSA): it can be used in cases of OSA / OHS overlap, but the situation here is more suggestive of
OHS. Long-term oxygen therapy (LTOT) alone should not be used here as without additional ventilatory support oxygen therapy
may exacerbate type II respiratory failure.

My Masterclass
Create exam

My history

My details

Q97.

A 43-year-old Portugese man presents with a 2 week history of feeling short of breath, fever and a rash. He is found to be
hypoxic with a PO2 of 6 on air but with a few sparse crackles. He has a rash over his back and an abnormal chest radiograph (see
images). What is the most likely pathogen?

See image

Cryptococcus
B

Mycobacterium tuberculosis
C

Pneumocystis carinii
D

Streptococcus
E

cytomegalovirus

The skin lesions are characteristic of Kaposi’s sarcoma and the chest radiograph (CXR) suggestive of a perihilar infiltrate/alveolar
shadowing.
This patient is likely to have a human immunodeficiency virus related disease.

The likeliest cause for this patient's LRTI is pneumocystis given the clinical history, examination and CXR.

Q98.

A 62-year-old woman was admitted with a two-month history of progressive breathlessness and weight loss. A chest
radiograph revealed a large right-sided pleural effusion and an intercostal drain was inserted. About 2 litres of fluid was
drained in the first 24 hours, and drain output in the next 24 hours was about 500ml, but then the drain stopped swinging
and a blockage was suspected.

What is the correct management?

A
Chest X-ray
B
Flush chest drain
C
Pull chest drain 2cm further out
D
Push chest drain 2cm further in
E
Remove chest drain

A blocked drain is a common problem. Every chest drain should be swinging as this indicates the correct positioning in the
pleural space. A drain can easily get blocked by debris. This is why regular pleural saline flushes are typically prescribed to
prevent this from occurring. If the drain is still not swinging after flushing it, then a chest x-ray is indicated.

Q99.
A 46-year-old woman with long-standing asthma was seen in the respiratory outpatient clinic with increased wheeze, cough and
sputum production. She was found to have an elevated serum total IgE and moderate eosinophilia. A high-resolution (HR) CT
showed central bronchiectasis with mucus plugging.

Which test would help confirm a diagnosis of allergic bronchopulmonary aspergillosis (ABPA)?

A
fungal hyphae seen in bronchial lavage fluid
B
negative Aspergillus immunoglobulin G (IgG)
C
positive ANCA
D
positive Aspergillus-specific IgE
E
positive serum β-glucan

A diagnosis of allergic bronchopulmonary aspergillosis (ABPA) with bronchiectasis is made based upon an elevated total IgE,
eosinophilia, positive Aspergillus-specific IgE or skin-prick test, positive Aspergillus immunoglobulin G (IgG) / precipitins and CT
findings of central bronchiectasis. β-Glucan is a pan-fungal marker that is detected in serum in cases of invasive fungal disease.

Q100.

A 35-year-old woman known to suffer from rheumatoid arthritis gives a history of cough with copious phlegm with
intermittent haemoptysis. She also gets recurrent chest infections. She is on weekly methotrexate along with a non-
steroidal anti-inflammatory drug (NSAID). Her chest radiograph shows linear radioluciences at both bases. A high-
resolution computed tomography scan (HRCT) of the chest confirms bronchiectasis. What is the most likely cause?

A
Cystic fibrosis.
B
Foreign body aspiration
C
Kartagener's syndrome
D
Old tuberculosis
E
Rheumatoid arthritis
E

Bronchiectasis may be due to many causes. Post infection (TB, measles, whooping cough etc), foreign body aspiration,
adenomas, hypogammaglobulinaemia, Kartagener's syndrome, Young's syndrome, cystic fibrosis, allergic bronchopulmonary
aspergillosis are the main causes. Up to 4 % of patients with rheumatoid arthritis develop bronchiectasis.

Q101.

A 19-year-old university student presented as an emergency to hospital with left-sided pleuritic chest pain and breathlessness.
She had no past medical history and took no regular medication. She smoked 20 cigarettes daily. A chest radiograph revealed a
50% left pneumothorax.

What is the most appropriate initial management?

A
aspiration of the left pleural cavity followed by repeat chest radiograph
B
chest drain insertion and admission for observation
C
chest drain insertion and discharge home with a one-way drainage (Heimlich) valve
D
referral for thoracoscopic left pleurectomy and pleurodesis
E
smoking cessation advice

In a young, previously fit person pleural aspiration may be all that is required to relieve symptoms and is the initial treatment of
choice. Surgery is usually reserved for patients with recurrent pneumothorax. Smoking cessation advice, while highly relevant, is
not the most appropriate initial management option.
Q102.

A 40-year-old female is breathless and has a full set of lung function performed: FEV1 1.1L (60%) FVC 1.3 L (55%) FEV1/FVC ratio
= 84% TLC = 65% predicted RV = 58% predicted TLCO = 55% predicted KCO = 105% predicted What is the most likely diagnosis?

A
Asthma
B
COPD
C
Fibrosing alveolitis
D
Obesity
E
Obliterative bronchiolitis

The lung function tests show a significant restrictive defect. Only obesity or a fibrosing disease may fit this picture given the
normal/high KCO (i.e. after correcting for alveolar volumes). The most likely answer is obesity, as the gas exchange after
correcting for the alveolar volume (TLCO) would tend to be low in fibrosing alveolitis. Both COPD and obliterative bronchiolitis
show an obstructive picture but with low TLCO whereas asthma will also produce an obstructive pattern but with normal or high
TLCO.

Q103.

A 55-year-old man known to suffer from rheumatoid arthritis (RA) is admitted with a left pleural effusion. He has never smoked
and apart from dyspnea, he is asymptomatic. Which of the following is true??

A
In pleural effusion secondary to RA, pleural pH is usually >7.35.
B
Rheumatoid arthritis is unlikely to be the cause of the effusion if pleural fluid glucose is above 1.6mmol/L.
C
Rheumatoid pleural effusions have high C4 complement levels.
D
The majority of patients with rheumatoid pleural effusions are women.
E
Up to 40% of patients with rheumatoid arthritis develop pleural effusions.

Pleural involvement occurs in 5% of patients with rheumatoid arthritis. It usually affects men and it can be serous, turbid,
yellow, milky or haemorrhagic. RA is unlikely to be the cause of a pleural effusion if the pleural fluid glucose is more than
1.6mmol/L. Eighty percent of rheumatoid pleural effusions have a pleural fluid pH of less than 7.30. In rheumatoid pleural
effusions, C4 complement level is below 0.04 g/L.

Q104.

A 35 year old woman was referred to the respiratory clinic with poorly controlled asthma. She was taking Symbicort 200/6 2
puffs BD and was using an entire canister of Salbutamol every month. On systems review she reported perennial rhinitis and a
decreased sense of smell. She worked on the front desk in a bakery. She kept 2 cats at home. She reported an allergy to non
steroidal anti-inflammatory drugs, which made her asthma worse.

What would be the next step in her management?

A
Advise her to change her job
B
Advise her to re-home the cats
C
Increase dose of inhaled corticosteroid
D
Trial of leukotriene receptor antagonist
E
Trial of oral antihistamine

The loss of sensation of smell suggests nasal polyps. Polyps, NSAID sensitivity and asthma are typical for Samter’s triad. Patients
often respond to leukotriene receptor antagonists, which improve both asthma and sinus symptoms. NSAIDS should be avoided.
There is no reason for her to change jobs or re-home her cats. Antihistamines are not useful in this condition. The aspirin
sensitivity is not IgE mediated.
My Masterclass

Create exam

My history

My details

Q105.

65 year old man presented to the Emergency Department with 4-week history of worsening breathlessness with a sudden
deterioration on the day of admission. He also complained of small volume hemoptysis (specs of blood) over the last few weeks.
A recent CT scan had demonstrated a large central lesion near the left main bronchus and a small left pleural effusion. He was
awaiting bronchoscopy for histological diagnosis and further management.

His past medical history was otherwise unremarkable, but he was a lifelong heavy smoker. His vital signs included pulse
120/min, BP 150/70 mmHg, SaO2 91% on 6 liters of O2, temperature 37.5oC and respiratory rate 28/min. On examination of the
chest he had dullness and reduced air entry on the left side. His ECG showed sinus tachycardia and CXR revealed complete
white-out of left hemithorax with the trachea deviated to the left.

What is the most likely diagnosis?

Collapse of left lung


B

Left pleural effusion


C

Myocardial infarction
D

Pneumothorax
E

Pulmonary embolus

It can be difficult to differentiate total collapse from massive pleural effusion as both conditions appear as white-out of
hemithorax. However, in massive pleural effusion the trachea will be deviated away from the side of the effusion while in total
collapse the trachea will be deviated towards the side of the collapse. This is important as the management of those two
conditions is different. This patient has suffered total collapse of left lung due to compression of left main bronchus from the
central lesion, which can explain the sudden deterioration. He almost certainly has a pleural effusion as well (mild on recent CT
scan), but it is not massive as the trachea is deviated towards the side of the effusion. He certainly has risk factors for Pulmonary
embolus but his CXR points to another diagnosis.
Q106.

A 55 year old man presented to the Emergency Department in the evening with 4-hour history of right sided pleuritic chest pain
and shortness of breath. He denied any history of trauma to the chest. His past medical history included hypertension, diabetes
and ischaemic heart disease. He was a lifelong smoker.

On examination his vital signs included temperature 37.4oC, pulse 115/min, BP 130/70 mmHg, respiratory rate 28/min and
SaO2=90% on 35% oxygen. Auscultation of the chest revealed reduced air entry on the right side. An ECG showed sinus
tachycardia and a CXR confirmed a right sided pneumothorax (3cm, central trachea).

What is the best immediate management?

A
Admit for observation
B
Arrange chest ultrasound to mark site for chest drain insertion
C
Arrange insertion of chest drain under ultrasound guidance
D
Aspirate pneumothorax with cannula
E
Insert chest drain

This patient has presented with spontaneous pneumothorax. He is >50 years old and has a significant smoking history. The size
of the pneumothorax is >2cm and he requires 35% oxygen to maintain saturation. According to the guidelines he should be
treated with chest drain immediately. As per guidelines the size of the chest drain should be 8-14F.

My Masterclass

Create exam

My history

My details

Q107.
A 20 year old woman with cystic fibrosis had had three admissions with respiratory infections in the last 12 months. She
produced purulent sputum daily and was finding it difficult to cough this up. Her spirometry showed an FEV1 of 50% predicted
and FVC 70% predicted. Her current treatment was postural drainage and physiotherapy in the mornings, pancreatic
supplements, vitamins, inhaled salbutamol, and annual influenza vaccination.

Which would be the most appropriate addition to her treatment?

Inhaled long acting bronchodilator

Nasal corticosteroids

Nebulised recombinant DNase

Oral amoxicillin

Oral steroids

Nebulised recombinant DNase can enhance sputum clearance and improve lung function, but it is expensive and its benefit
should be assessed in each individual patient. Regular inhaled corticosteroids may be helpful in cystic fibrosis but oral steroids
should be avoided. Long term use of macrolide antibiotics such as azithromycin, or nebulised antibiotics such as colomycin or
tobramycin, may reduce the frequency of exacerbations.

Q108.

A 55 year old White British man with a history of alcohol abuse presented to hospital with a two week history of cough and right
sided pleuritic chest pain. Vital signs included temperature 38.1oC, pulse 110/min, BP 130/75 mmHg, and Sats 97% (on air).
Examination of the chest revealed a dull percussion note and decreased air entry at the right base.

Chest x-ray confirmed a moderate right sided pleural effusion, and a diagnostic pleural tap was performed by the admitting
team under direct ultrasound guidance.

Pleural fluid results were as follows: Microscopy - Gram stain negative (culture results awaited)
Protein - 30g/L
Glucose - 2.0 mmol/L pH - 7.1
What is the most appropriate treatment?

A
Antibiotics for community acquired pneumonia (intravenous)
B
Antibiotics for community acquired pneumonia (intravenous) and for tuberculosis (oral)
C
Antibiotics for community acquired pneumonia (intravenous) and intercostal chest drain
D
Antibiotics for community acquired pneumonia (oral)
E
Antibiotics for community acquired pneumonia (oral) and for tuberculosis (oral)

This man has an empyema which requires prompt treatment with IV antibiotics and drainage. Thoracic surgical input may be
required if it not resolve. Empyema is more common in the immunocopromissed and alcoholics. A pleural effusion with a pH of
less than 7.2 requires a chest drain as per British Thoracic Society guidelines.

Q109.

A 25-year-old gentleman with a history of asthma attends the casualty department with a history of acute dyspnoea. He is
hypoxic with a PaO2 of 7.8 Kpa. The chest radiograph is shown (see image). What is the appropriate management?
See image
A

Admission to ITU and ventilatory support

Anti-coagulation and urgent spiral CT thorax

Continuous positive airway pressure (CPAP)

High flow oxygen, nebulised bronchodilators and oral steroids

None of the above.

The chest radiograph shows a right pneumothorax. He needs a needle aspiration and may need an intercostal chest drain if
needle aspiration is not successful.
Q110.

A 56y stockbroker with a 40 pack-year history of smoking had a diagnosis of chronic obstructive pulmonary disease made on the
basis of spirometric findings. He was referred to a general medical clinic with a 9 month history of progressive breathlessness
and a dry cough. His GP had started inhaled steroids and anti-cholinergics with modest benefit.

On examination he was thin, without skeletal deformity, and was mildly hyperinflated with poor chest expansion. On
auscultation he had soft breath sounds and bi-basal fine inspiratory crackles. A chest X-ray showed prominent hila. Lung
function tests showed FEV1=1.25L (35% predicted), FVC=2.2L (65%), PFR=350L/min (85%), FEV1/FVC Ratio=58%, TLC=120%, RV=
90%, corrected gas exchange 50% predicted.

What is the most likely diagnosis?

A
Chronic obstructive pulmonary disease
B
Idiopathic pulmonary fibrosis
C
Idiopathic pulmonary fibrosis and chronic obstructive pulmonary disease
D
Primary pulmonary hypertension
E
Pulmonary sarcoidosis

This picture is of a mixed obstructive and restrictive defect. The degree of obstruction is moderately severe yet the lung volumes
are smaller than expected. This in conjunction with the progressive breathlessness and bibasal crackles make dual pathology
likely, with COPD masking fibrosis. Sarcoidosis could present similarly, but rarely causes obstruction.

Q111.

A 74-year-old man was treated in hospital for exacerbation of Chronic Obstructive Pulmonary Disease (COPD). His condition has
improved significantly and he is keen to go home. His repeated arterial blood gas (ABG) reading (on air) showed a pH of 7.35,
pCO2 of 4.5kPa, pO2 of 7.1kPa, HCO3 of 26 mmo/L. What action would be most appropriate?

A
Discharge the patient and advise a follow up by his GP.
B
Discharge the patient and arrange a follow up in 6 weeks' time with a repeated ABG.
C
Discharge the patient once his ABG returns to normal.
D
Discharge the patient, prescribing an oxygen cylinder.
E
Request an oxygen concentrator and discharge the patient once an oxygen concentrator is fitted.

Long-term oxygen treatment is recommended if pO2 is less than 7.3kPa on two occasions in remission of COPD. This patient's
p02 is likely to improve when he recovers from his current exacerbation and his ABG should be re-checked in about 6 weeks
time. As he seems able to tolerate his mild hypoxia, he would not require any oxygen supplementation at home at this stage.

Q112.

A 55-year-old woman is referred with a 10-day history of productive cough, fever and increasing shortness of breath. Her chest
radiograph shows:
See image
A

anomalous superior vena cava.

right upper lobe collapse

right upper zone consolidation

superior mediastinal mass

widened mediastinum

B
Note also the tenting of the right hemidiaphragm and the raised right hilum.

Depending on the clinical context the commonest causes to consider would be infection, carcinoma of the lung and foreign
body. Other tumours, e.g. carcinoid, benign adenoma, and pulmonary infarct would be much less likely.

Q113.

A 51-year-old woman was admitted with sudden onset left sided pleuritic chest pain accompanied by shortness of breath. Her
only past medical history was of asthma, which had been well controlled on a low dose of inhaled corticosteroids, and ligation of
varicose veins, performed 2 weeks ago. Vital signs were pulse 130/min and BP 142/82 mmHg. On auscultation her chest was
clear. Arterial blood gases (on room air) were as follows: pH 7.34, p02 7.6kPa, pCO2 3.5 kPa. She was started on high flow
oxygen. A chest radiograph was normal. What would be the most appropriate immediate action?

A
Give salbutamol 5mg by nebulizer
B
Give treatment dose low molecular weight heparin
C
Request chest radiograph in expiration
D
Request CT pulmonary angiogram
E
Request D-dimer

The symptoms and findings clearly point towards pulmonary embolism (PE). As the clinical probability of PE is high,
measurement of D-dimer should not be performed, since the result would not alter the need for definitive investigation.
Measurement of D-dimer should only be performed when the probability of PE is low, when a normal value would be taken as
reassuring and further investigation would not be pursued. A CT pulmonary angiogram would be the imaging procedure of
choice to confirm the diagnosis, but the most appropriate immediate action would be to give a treatment dose of LMWH.

Q114.

A 60 year old woman presented with increasing breathlessness that had come on over a matter of weeks. She had no
other cardiac or respiratory symptoms, but on detailed history taking she revealed that her pet budgerigar had died 2
weeks earlier. On examination there were inspiratory crackles at both lung bases. A chest X ray showed upper lobe
changes, and a chest CT scan showed a mosaic pattern with a little peripheral honeycombing. Respiratory function tests
showed a moderate restrictive defect. Blood tests showed positive IgG avian antibodies but negative IgE antibodies.

Which would be appropriate management?


A

Clarithromycin - oral

Corticosteroids - inhaled

Long acting beta agonist - inhaled

Observation - with repeat respiratory function tests and chest X ray

Prednisolone - oral

In hypersensitivity pneumonitis removal from exposure is the most important part of the management, but in this case the
extent of the radiographic and respiratory function changes suggests that more intervention is needed. Oral corticosteroids
speed up resolution of any reversible element of the lung involvement, but fibrosis may be established at the time of
presentation and irreversible. The optimum dose and duration of oral corticosteroid treatment are uncertain, but it is likely to
be needed for at least one to two months in such cases.

You might also like